0% encontró este documento útil (0 votos)
576 vistas260 páginas

Funciones Matemáticas: Guía Completa

Este documento proporciona una guía sobre funciones matemáticas. Incluye 13 secciones que describen diferentes tipos de funciones como funciones lineales, cuadráticas, polinómicas, racionales, exponenciales y logarítmicas. Explica sus características, gráficas y cómo resolver problemas relacionados con cada tipo de función. El objetivo es promover materiales educativos libres y de alta calidad sobre funciones.
Derechos de autor
© © All Rights Reserved
Nos tomamos en serio los derechos de los contenidos. Si sospechas que se trata de tu contenido, reclámalo aquí.
Formatos disponibles
Descarga como PDF, TXT o lee en línea desde Scribd
0% encontró este documento útil (0 votos)
576 vistas260 páginas

Funciones Matemáticas: Guía Completa

Este documento proporciona una guía sobre funciones matemáticas. Incluye 13 secciones que describen diferentes tipos de funciones como funciones lineales, cuadráticas, polinómicas, racionales, exponenciales y logarítmicas. Explica sus características, gráficas y cómo resolver problemas relacionados con cada tipo de función. El objetivo es promover materiales educativos libres y de alta calidad sobre funciones.
Derechos de autor
© © All Rights Reserved
Nos tomamos en serio los derechos de los contenidos. Si sospechas que se trata de tu contenido, reclámalo aquí.
Formatos disponibles
Descarga como PDF, TXT o lee en línea desde Scribd

FUNCIONS

Llibre de text

Gerard Romo Garrido


Toomates Coolección
Los documentos de Toomates son materiales digitales y gratuitos. Son digitales porque están pensados para ser consultados
mediante un ordenador, tablet o móvil. Son gratuitos porque se ofrecen a la comunidad educativa sin coste alguno. Los libros de
texto pueden ser digitales o en papel, gratuitos o en venta, y ninguna de estas opciones es necesariamente mejor o peor que las otras.
Es más: Suele suceder que los mejores docentes son los que piden a sus alumnos la compra de un libro de texto en papel, esto es un
hecho. Lo que no es aceptable, por inmoral y mezquino, es el modelo de las llamadas "licencias digitales" con las que las editoriales
pretenden cobrar a los estudiantes, una y otra vez, por acceder a los mismos contenidos (unos contenidos que, además, son de una
bajísima calidad). Este modelo de negocio es miserable, pues impide el compartir un mismo libro, incluso entre dos hermanos,
pretende convertir a los estudiantes en un mercado cautivo, exige a los estudiantes y a las escuelas costosísimas líneas de Internet,
pretende pervertir el conocimiento, que es algo social, público, convirtiéndolo en un producto de propiedad privada, accesible solo a
aquellos que se lo puedan permitir, y solo de una manera encapsulada, fragmentada, impidiendo el derecho del alumno de poseer
todo el libro, de acceder a todo el libro, de moverse libremente por todo el libro.
Nadie puede pretender ser neutral ante esto: Mirar para otro lado y aceptar el modelo de licencias digitales es admitir un mundo más
injusto, es participar en la denegación del acceso al conocimiento a aquellos que no disponen de medios económicos, y esto en un
mundo en el que las modernas tecnologías actuales permiten, por primera vez en la historia de la Humanidad, poder compartir el
conocimiento sin coste alguno, con algo tan simple como es un archivo "pdf". El conocimiento no es una mercancía.
El proyecto Toomates tiene como objetivo la promoción y difusión entre el profesorado y el colectivo de estudiantes de unos
materiales didácticos libres, gratuitos y de calidad, que fuerce a las editoriales a competir ofreciendo alternativas de pago atractivas
aumentando la calidad de unos libros de texto que actualmente son muy mediocres, y no mediante retorcidas técnicas comerciales.
Este documento se comparte bajo una licencia “Creative Commons 4.0 (Atribution Non Commercial)”: Se permite, se promueve
y se fomenta cualquier uso, reproducción y edición de todos estos materiales siempre que sea sin ánimo de lucro y se cite su
procedencia. Todos los documentos se ofrecen en dos versiones: En formato “pdf” para una cómoda lectura y en el formato “doc”
de MSWord para permitir y facilitar su edición y generar versiones parcial o totalmente modificadas.

¡Libérate de la tiranía y mediocridad de las editoriales! Crea, utiliza y comparte tus propios materiales didácticos

Toomates Coolección Problem-solving:


Geometría Axiomática , Problemas de Geometría , Introducción a la Geometría , Álgebra
Teoría de números , Combinatoria , Probabilidad , Trigonometría , Desigualdades
Números complejos .

Toomates Coolección Llibres de Text (En catalán):


Nombres (Preàlgebra) , Àlgebra , Proporcionalitat , Geometria analítica
Compendium ACM4 , Combinatòria i Probabilitat , Estadística , Trigonometria
Funcions , Nombres Complexos , Àlgebra Lineal 2n batx. , Geometria Lineal 2n batx.
Càlcul Infinitesimal 2n batx. , Programació Lineal 2n batx .

Recopilaciones de Pruebas PAU: Catalunya TEC , Catalunya CCSS , Galicia , País Vasco ,
Portugal A , Portugal B

Toomates Coolección Compendiums:


AIME , AMC 8 , AMC 10 , AMC 12 , Archimede , Canguro , Cangur pdf , Cangur doc
HMMT , IGO , IMO , Kangourou , Kangaroo , Mathcounts , OME , OMEFL , OMEC
OMEM , OMI , SMT , USAMO

¡Genera tus propias versiones de este documento! Siempre que es posible se ofrecen las versiones
editables “MS Word” de todos los materiales, para facilitar su edición. Descarga en los siguientes enlaces
la versión ".doc" de este documento en:
www.toomates.net/biblioteca/Funcions01.doc → www.toomates.net/biblioteca/Funcions14.doc

¡Ayuda a mejorar! Envía cualquier duda, observación, comentario o sugerencia que tengas a
[email protected]

¡No utilices una versión anticuada! Todos estos documentos se mejoran constantemente. Descarga
totalmente gratis la última versión de los documentos en los correspondientes enlaces superiores, en
donde siempre encontrarás la versión más actualizada.

Encontrarás muchos más materiales para el aprendizaje y enseñanza de las matemáticas en la web
www.toomates.net

Versión de este documento: 24/04/2022


Índex.
1 Les funcions reals de variable real. → Arxiu en format .doc

1.1 Característiques principals de les funcions.


1.2 Operacions amb funcions.
1.3 Composició de funcions.
1.4 La funció inversa.
2 Les funcions lineals (f. polinòmiques de primer grau). → Arxiu en format .doc

2.1 Característiques de la funció de primer grau.


2.2 La gràfica de les funcions de primer grau.
2.3 Determinació de funcions de primer grau.
2.4 Punt d'intersecció de funcions de primer grau.
2.5 Funcions lineals en context.
3 Les funcions de segon grau. → Arxiu en format .doc

3.1 Funcions de segon grau en forma general.


3.2 Funcions de segon en forma estàndard.
3.3 Determinació de funcions de segon grau (amb un punt en l’eix Y).
3.4 Determinació de funcions de segon grau (cas general).
3.5 Punts de tall amb funcions de segon grau.
3.6 Desigualtats amb funcions de segon grau.
3.7 Transformació de funcions de segon grau.
3.8 Funcions de segon en forma factoritzada.
3.9 Funcions de segon grau en context.
4 Les funcions polinòmiques de tercer grau i superior. → Arxiu en format .doc

4.1 Característiques de les funcions polinòmiques.


4.2 La gràfica de les funcions polinòmiques.
4.3 Punts notables d'una funció polinòmica.
4.4 Funcions polinòmiques presentades en forma factoritzada.
4.5 Determinació de f. polinòmiques donats els punts de tall amb l'eix X.
4.6 Transformacions de funcions polinòmiques.
4.7 Repàs de funcions polinòmiques.
5 Les funcions racionals. → Arxiu en format .doc

5.1 Les funcions de proporcionalitat inversa.


5.2 Domini de definició d’una funció racional.
5.3 Punts de tall amb l'eix X.
5.4 Simplificació de funcions racionals.
5.5 Asímptotes verticals.
5.6 Indeterminació 0/0. Discontinuïtats evitables.
5.7 Comportament d’una funció racional en l’infinit.
5.8 Estudi d’una funció racional (sense derivació).
5.9 Problemes PAU amb funcions racionals (sense derivació).
6 Les funcions radicals. → Arxiu en format .doc

6.1 Característiques de les funcions radicals.


6.2 La gràfica de les funcions radicals.
6.3 Domini de definició de les funcions radicals.
6.4 Punts notables de la gràfica de funcions amb arrels quadrades.
6.5 Arrels quadrades com a parametritzacions de la circumferència.
6.6 Transformacions de la funció arrel quadrada.
6.7 Transformacions de funcions amb arrels cúbiques.
6.8 Repàs d’equacions i funcions amb arrels quadrades (I).
6.9 Repàs d’equacions i funcions amb arrels quadrades (II).
6.10 Repàs d’equacions i funcions amb arrels quadrades (III).
7 Les funcions exponencials. → Arxiu en format .doc

7.1 Característiques de les funcions exponencials.


7.2 La gràfica de les funcions exponencials.
7.3 Determinació de funcions exponencials.
7.4 Modelitzant amb funcions exponencials.
7.5 Transformacions de funcions exponencials.
7.6 Punts de tall amb funcions exponencials.
7.7 Repàs de funcions exponencials.
8 Les funcions logarítmiques. → Arxiu en format .doc

8.1 Característiques de les funcions logarítmiques.


8.2 La gràfica de les funcions logarítmiques.
8.3 Transformacions de funcions logarítmiques.
9 Les funcions trigonomètriques. → Arxiu en format .doc

10 Les funcions definides a trossos. → Arxiu en format .doc

10.1 Concepte de funció definida a trossos.


10.2 La funció part entera.
11 La funció valor absolut. → Arxiu en format .doc

11.1 El concepte de valor absolut.


11.2 Transformacions de la funció valor absolut.
12 Problem-Solving amb funcions. → Arxiu en format .doc

12.1 El llenguatge de les funcions.


12.2 Problem-Solving amb la gràfica de la funció.
13 Llistes de repàs amb funcions. → Arxiu en format .doc

13.1 Gràfiques Segon grau-Arrel quadrada-exponencial-logarítmica.


Solucions. → Arxiu en format .doc

La continuïtat natural d’aquest volum és “Càlcul infinitesimal”:


http://www.toomates.net/biblioteca/Calcul.pdf
1 Funcions reals de variable real.
1.1 Definicions i característiques principals de les funcions.
Definició de funció.
Una funció f : A  B és una correspondència que assigna a cada element a de
A un (i només un) element del conjunt B, que denotem per mitjà de f (a) . Es
diu que f (a) és la imatge de a per f .

Si A  IR i B  IR direm que f és una funció real de variable real.

Exemples de funcions: Projeccions.

a) Donades dues rectes A i B, i un punt O exterior a totes dues, podem definir


una funció f : A  B de la següent forma: Per a cada punt P de A, definim la
sema imatge f (P) com el punt d’intersecció entre les rectes OP i B:

b) Donada una recta A i una circumferencia B, podem definir una funció


f : A  B de la següent forma: Per a cada punt P de A, definim la sema imatge
f (P) com el punt d’intersecció entre les rectes OP i B, on O és el centre de la
circumferència.
Formes de presentar una funció.

Un enunciat o regla ens informa amb paraules sobre la forma en què es


relacionen entre elles les variables. Per exemple: «El nombre de subscriptors a la
nostra revista creix uniformement des de l'any 2005..."

Una taula associa a determinats valors d’una de les variables els corresponents
valors
de l’altra. Per exemple:

Una gràfica representa de forma clara i visual la relació que hi ha entre les
variables.

Una fórmula o expressió algebraica, quan això és possible, la dependència


exacta
que es dona entre variables; és a dir, com mitjançant unes determinades
operacions
realitzades sobre una d’elles, en podem obtenir l’altra. Per exemple:

N ( x)  21x  41935

1.1.1
Dóna l’expressió algèbrica d’una funció que tingui aquesta taula:
Domini i recorregut.

El conjunt A on té sentit la funció, s'anomena domini de f . Es representa per


Dom f .
Per exemple, x  2 no pertany al domini de la funció f ( x)  x perquè no
existeix el nombre f (2)   2 .

Direm que un nombre b és imatge d'una funció f quan b  f (x) per a cert x
del domini.
Per exemple: 21 és imatge de f ( x)  x 2  4 x perquè f (7)  72  4  7  21 .

El recorregut d'una funció és el conjunt de totes les imatges. Es representa per


Im f . També s'anomena conjunt imatge. És difícil veure quin és el recorregut a
partir de l'equació d'una funció. El millor és fer-ho a partir de la gràfica i
projectar-la sobre l'eix y. El recorregut serà l'interval de l'eix y que queda pintat
per la gràfica. El recorregut de la funció és el conjunt de valors que adopta la
funció.

La gràfica d’una funció ens permet veure, en un mateix dibuix, el domini de la


funció (eix d’abscisses, X) i les imatges (eix d’ordenades, Y ). Ens dóna molta
informació sobre el comportament de la funció en tot el seu domini.

1.1.2
Determineu el domini de les següents funcions.
1 1 x
a) f ( x)  2 x 2  x 3 b) f ( x)  c) f ( x) 
x4 x2  2x
1
d) f ( x)  x  1 e) f ( x)  x  1  x f) f ( x) 
x2  9
Característiques singulars de la gràfica d'una funció.

Simetries.
Direm que una funció f és parella si f ( x)  f ( x) .

Direm que una funció f és imparella o senar si f ( x)   f ( x)

Periodicitat.
Direm que una funció f és periòdica de període T  0 si la seva gràfica
es va repetint en intervals de longitud T, és a dir, si f x  n k   f (x)

Monotonia.
Direm que una funció f és creixent si la seva gràfica, mirada d’esquerra
a dreta, va cap a munt, és a dir:

x  y  f x   f (x) (creixent)
x  y  f x   f (x) (estrictament creixent)

Direm que una funció f és decreixent si la seva gràfica, mirada


d’esquerra a dreta, va cap avall, és a dir:

x  y  f x   f (x) (decreixent)
x  y  f x   f (x) (estrictament decreixent)
Punts notables de la gràfica d’una funció.

Punts de tall amb l'eix X.


Són els valors del domini per als què f ( x)  0

Punt de tall amb l'eix Y.


És el valor (si existeix) de f (0) .

Punts de tall entre dues gràfiques.

Són les possibles solucions de l'equació f ( x)  g ( x)


1.1.3
Donada la següent gràfica:

a) Domini de definició.
b) Imatge de la funció.
c) Punts de tall amb l'eix X.
d) Punt de tall amb l'eix Y.
e) Determina f (4) .
f) Resol f ( x)  1 .

1.1.4
Donada la següent gràfica d'una funció, determina:
a) f (2) , f (0) , f (2) , f (3)
b) El domini i el recorregut de la funció.
c) Punts de tall amb l'eix X (aproximadament)
d) Punt de tall amb l'eix Y.
e) Els valors de x per als què f ( x)  3 .
f) El mínim i el màxim de la funció.
1.1.5
Donada la següent gràfica d'una funció, determina:
a) f (4) , f (2) , f (0) , f (4) .
b) El domini i el recorregut de la funció.
c) Punts de tall amb l'eix X (aproximadament).
d) Punt de tall amb l'eix Y.
e) El mínim i el màxim de la funció.

1.1.6
Donades les següents gràfiques, determina:
a) f (0) , g (3) .
b) El domini i el recorregut de f (x) .
c) El domini i el recorregut de g (x) .
d) Punts de tall de g (x) amb l'eix X.
e) Punts de tall entre f (x) i g (x) .
1.1.7
Donades les següents gràfiques, determina:
a) f (3) , g (3) , g (4) , f (4) .
b) El domini i el recorregut de f (x) .
c) Punts de tall entre f (x) i g (x) .
1.1.8
Anàlisi d'una gràfica d'un diari.

Font: El Punt-Avui 11/04/2012

Espanya i Alemanya necessiten emetre bons a deu anys de forma regular per finançar-se. Si la
rendibilitat dels bons espanyols a deu anys és del 6,099%, i la dels bons alemanys del 2,621%, la
diferència és del 3,478%, és a dir, de 347 punts bàsics. En aquest cas, la prima de risc espanyola
seria de 347 punts. La prima de risc dels països de la Unió Europea es calcula respecte de
Alemanya perquè se suposa que el seu deute públic és el que menor risc de impagament té, fins i
tot se sol acceptar que el seu risc d'impagament és 0.

1. Construeix una gràfica amb aquestes dades, però amb l’eix horitzontal
complet, és a dir, on apareguin també tots els dies que falten a la gràfica del
diari.

2. Respon a les següents preguntes:


a) En quins dies s’ha observat una prima de risc per sobre de 350 punts?
b) En quins dies s’ha observat una prima de risc per sota dels 345 punts?
c) En quins dies s’ha observat una prima de risc entre 355 i 270 punts?
d) Quina ha sigut la variació de la prima de risc entre el 3 i el 10 d’Abril?
e) Determina els intervals on observem un comportament creixent i un
comportament decreixent de la prima de risc.
f) Determina les dies on s’han localitzat mínims i màxims. Quin és el mínim
absolut i el màxim absolut? En quins dies s’han localitzat?
1.2 Operacions amb funcions.

Suma, resta, multiplicació i divisió de funcions.


Les funcions es poden sumar, restar, multiplicar i dividir, operant les expressions
corresponents.

Suma de funcions: ( f  g )( x)  f ( x)  g ( x)

Resta de funcions: ( f  g )( x)  f ( x)  g ( x)

Multiplicació de funcions: ( f  g )( x)  f ( x)  g ( x)

f f ( x)
Divisió de funcions:  ( x)  si g ( x)  0
g g ( x)

Domini de definició de les operacions amb funcions.


Observem que l'únic problema afegit de domini el trobarem amb la divisió,
perquè no estarà definida quan el denominador sigui zero.

Exercici resolt.

Donades f ( x)  x 2  1 i g ( x)  3x  5 , determina
f
a)  f  g (1) b) f  g (3) c) ( f  g )(5) d)  0 
g

Solució:
a)
f (1)  2
   f  g  (1)  f (1)  g (1)  2  8  10
g (1)  8 

b)
f (3)  10 
   f  g (1)  f (3)  g (3)  10  (4)  14
g (3)  4

c)
f (5)  26
   f  g (5)  f (5)  g (5)  26  20  520
g (5)  20 

d)
f (0)  1  f 
   0 
f (0) 1

g (0)  5  g  g (0) 5
Exercici resolt.
Donades f ( x)  8x  9 i g ( x)  2 x  1 , determina l'expressió algèbrica de les
següents funcions i el seu domini:
f
a)  f  g (x) b)  f  g (x) c) ( f  g )( x) d)   x 
g

Solució:
a)  f  g ( x)  f ( x)  g ( x)  8x  9  2 x  1
b)  f  g ( x)  f ( x)  g ( x)  8x  9  2 x  1
c) ( f  g )( x)  f ( x)  g ( x)  8x  9 2 x  1
f f ( x) 8 x  9
d)  x   
g g ( x) 2x  1

Domini: el domini de g ( x)  2 x  1 és  1/ 2, , i com que la suma, resta i


producte de funcions no altera el domini, el domini de  f  g (x) ,  f  g (x) , i
( f  g )( x) serà aquest mateix:  1/ 2, .
f
Ara bé, la funció  x  
f ( x)
tampoc estarà definida si el denominador és
g g ( x)
zero:
1
0  g ( x)  2 x  1  2 x  1  0  x  .
2
f
Per tant, el domini de  x   és  1/ 2, .
f ( x)
g g ( x)

1.2.1
Donades les gràfiques de f (x) i g (x) :

determina:
g
a) ( f  g )(4) b) g  f (0) c) ( f  g )(1) d)  (0)
f
f
e)  (0)
g
1.2.2
Donada la següent taula de valors X-Y:

determina:
f
a) ( f  g )(4) b)  f  g (2) c) ( f  g )(1) d)  (0)
g
1.3 Composició de funcions.

Definició de composició de funcions.

Les funcions es poden composar, és a dir, encadenar la sortida d'una a l'entrada


d'una altra. Per exemple:

f ( x )  3 x  2
  g  f ( x)  g  f ( x)   3x  2  5  9 x  12 x  9
2 2

g ( x)  x  5 
2

L'expressió g  f (x) es llegeix g composta amb f de x.

- Recorda sempre que per avaluar g  f (x) es comença per la funció de la


dreta, perquè és la primera que actua sobre la variable x.

- No has de confondre el símbol de composició "  " amb el multiplicació "·"

Si interpretem les funcions com màquines que tenen una entrada i una sortida:

La composició de funcions és enganxar màquines en cadena: La sortida d'una a


l'entrada de l'altra:

Podem enganxar tantes màquines (funcions) com volguéssim, una darrera l'altra:
Sobre la no commutativitat de la composició de funcions.

Observa que la composició de funcions en general no és commutativa:

 g  f  ( x)   f  g  ( x)
Per exemple: f ( x)  3x  2 , g ( x)  x 2  5
g  f ( x)  g  f ( x)  3x  22  5  9x 2  12x  9
 f  g ( x)  f g ( x)  3x 2  5  2  3x 2  15  2  3x 2  17

En la vida real, el cas més conegut de no commutativitat de funcions és el fer-se


un got de Colacao. El resultat no és el mateix si aboques primer la llet i després
el Cola-Cao que si primer aboques el cola-Cao al got buit i després la llet,
perquè et queda tot ple de grumets. Tanmateix, això no passa amb el Nesquik. Ni
passa amb la llet calenta. No hi ha commutativitat només amb Colacao i amb llet
freda.

f g

g f
Domini de la composició de funcions.
Per determinar domini de g  f hem de trobar els valors de x que compleixen
que:
- x és en el domini de f
- f (x) és en el domini de g.

Exercici resolt.
4
Donades f ( x)  2 x  1 i g ( x)  , determina
x 1
a)  f  g (2) b) g  f (3)

Solució:
  4  f (4)  2  4  1  7   f  g (2)  f g 2  7
4 4
a) g (2) 
2 1 1
b)
1 1
 g  f (3)  g  f  3 
4 4
f (3)  2(3)  1  7  g (7)   
 7 1  8 2 2

Molta atenció amb el tatxar operacions!


Si f ( x)  x 2 i g ( x)  x , llavors  f  g ( x)   x
2
no és x ,  f  g ( x)  x
g  f ( x)  x 2  x , i Domg  f   0,
Exercici resolt.
Amb les funcions f ( x)  4 x  1 i g ( x)  2 x 2  5x , comprova que, en general,
g  f ( x)   f  g ( x) :
Solució:
g  f ( x)  g ( f ( x))  g (4 x  1)  2(4 x  1)2  5(4 x  1) 
 2(16 x 2  8 x  1)  20 x  5  32 x 2  16 x  2  20 x  5  32 x 2  36 x  7
 f  g ( x)  f ( g ( x))  f (2 x2  5x)  4(2 x 2  5x)  1  8x 2  20 x  1

I clarament : 32 x2  36 x  7  8x2  20 x  1

1.3.1
Donades les funcions f ( x)  x 2  2 x i g ( x)  3  x , resol les següents
equacions:
a)  f  g ( x)  0 b) g  f ( x)  x 2  5  0

1.3.2
Donades f ( x)  x 2  9 i g ( x)  2 x  5 , resol la inequació f g x   0
Composició de funcions i translacions.

Donat un nombre k  0

Translació vertical cap a dalt: Translació vertical cap a baix:


f ( x)  g ( x )  k f ( x)  g ( x )  k

Translació horitzontal cap a l'esquerra: Translació horitzontal cap a la dreta:


f ( x)  g ( x  k ) f ( x)  g ( x  k )

...i la composició de les dues anteriors: f ( x)  g ( x  k )  q

Vegeu exercici 3.6.1 (Translacions amb funcions de segon grau)


Composició de funcions: Dilatacions i contraccions.

Donat un nombre k  0

Dilatació vertical: k  1 Contracció vertical k  1

f ( x)  k g ( x) f ( x)  k g ( x)

Contracció horitzontal k  1 Dilatació horitzontal k  1

f ( x)  g ( k x) f ( x)  g ( k x)

La composició de funcions i la seva interpretació geomètrica és clau per a


resoldre problemes algèbric complicats i no ofegar-nos en un mar d'àlgebra.
Estudia, per exemple, l'exercici 8.3.5.
Composició de funcions: Simetries.

Simetria horitzontal:
f ( x)  g (  x)

Simetria vertical:
f ( x )   g ( x)

Simetria central:
f ( x)   g (  x)
1.4 La funció inversa.

Definició de funció inversa.


Donada una funció f , anomenem inversa de f, i escriurem f 1 , a qualsevol
funció tal que
f  f 1  f 1  f  id

On id és la funció identitat: id ( x)  x , la funció "que no fa res".

Amb la interpretació de màquines de l'apartat 1.3, donada una màquina f ,


volem trobar una màquina f 1 que desfaci allò que fa f :

Només les funcions bijectives tenen inversa.

Amb les funcions, "elevar a la menys ú" no és dividir.


1 1
Amb els nombres, a 1  , per exemple: 4 1   0.25
a 4
Però amb les funcions, "elevar a la menys ú" es fer la inversa, no dividir.

1.4.1
Donades les funcions
x8
f ( x)  3 x  8 g ( x) 
3

Determina f  g i g  f . Què observes?

Inversa d'una composició de funcions.


Si f es posar-me la sabata, f 1 és treure'm la sabata.
Si q es posar-me el mitjó, q 1 és treure'm el mitjó.
Llavors  f  g  és primer posar-me el mitjó i després, a sobre, posar-me la
sabata.
 
La seva inversa és g 1  f 1 , és a dir, primer treure'm la sabata i després
treure'm el mitjó.
 
El que no faré és f 1  g 1 , és a dir, intentar treure'm el mitjó sense treure'm la
sabata primer (comprova-ho!)
Amb aquesta idea al cap, la inversa de la composició de funcions és la
composició d'inverses, però invertint també l'ordre:
 f  g 1  g 1  f 1
Càlcul de la funció inversa.
1. Comprovarem si la funció és bijectiva.
La manera més fàcil de veure si una funció és bijectiva és fer-ho gràficament: Si
per qualsevol línia horitzontal que tracem sobre la gràfica aquesta talla la funció
només en un punt, la funció és injectiva. Si el recorregut són tots els reals, la
funció és exhaustiva, i si es donen aquests dos supòsits, la funció és bijectiva.

2. Aïllarem la y en funció de x.

3. Canviarem la x per la y a l'equació associada.

Exemple resolt.

Calcula la funció inversa de la funció f ( x)  x3  2 .

Solució:

1. Comprovem que és una funció bijectiva observant la seva gràfica

2. Aïllem la y en funció de x:

y  f ( x)  x 3  2  y  x 3  2  y  2  x 3  3 y  2  x

3. Canviarem la x per la y a l'equació associada:

y  f 1 ( x)  3 x  2
Interpretació gràfica de la funció inversa.

La gràfica d'una funció i la seva inversa són simètriques respecte la bisectriu del
1r i 3r quadrant, és a dir, respecte la recta y  x .

Amb l'exemple anterior:

Aquesta idea és molt útil per a aprendre les gràfiques de les funcions per
parelles: La gràfica de la funció i la de la seva inversa:
Una manera visual d’estudiar funcions inverses.

La història del concepte de funció

El concepte de funció es va establir cap al segle XVIII, encara que ja


anteriorment alguns matemàtics hi treballaven de manera intuïtiva. En l’obra
Introductio in analysin infinitorum, Leonhard Euler intenta per primera vegada
proporcionar una definició formal del concepte de funció en afirmar que: “Una
funció de quantitat variable és una expressió analítica formada de qualsevol
manera per aquesta quantitat variable i per nombres o quantitats constants”.
Aquesta definició difereix de l’actual, tal com es veurà, atès que set anys
després, en el pròleg de les Institucions del càlcul diferencial, va afirmar:
“Algunes quantitats en veritat depenen d’altres; si en ser combinades les
darreres, les primeres també sofreixen canvi, llavors les primeres es diuen
funcions de les últimes. Aquesta denominació és bastant natural i comprèn cada
mètode mitjançant el qual una quantitat pot ser determinada per unes altres.
Així, si x denota una quantitat variable, llavors totes les quantitats que depenen
de x en qualsevol forma estan determinades per x i se les anomena funcions
d’x”.

El matemàtic suís Leonhard Euler (1707-1783) és un personatge essencial en el


desenvolupament de les funcions perquè precisà el concepte de funció, va fer un
estudi sistemàtic de totes les funcions elementals, incloent-hi les seves derivades
i integrals; no obstant això, el concepte mateix de funció va néixer amb les
primeres relacions observades entre dues variables, fet que segurament va sorgir
en els inicis de la matemàtica, amb civilitzacions com la babilònica, l’egípcia i la
xinesa.

Abans d’Euler, el matemàtic i filòsof francès René Descartes (1596-1650) va


mostrar en els seus treballs de geometria que tenia una idea molt clara dels
conceptes variable i funció, fent una classificació de les corbes algebraiques
segons els seus graus, reconeixent que els punts d’intersecció de dues corbes
s’obtenen resolent, de manera simultània, les equacions que les representa.

Font: “Iniciació a les Matemàtiques per a l’enginyeria”


2 Les funcions lineals (funcions polinòmiques de primer
grau).
2.1 L’equació punt-pendent de les funcions lineals.

Ara és un bon moment per repassar...


Les equacions de primer grau. (Tema 8 del Llibre d'Àlgebra)
Les inequacions de primer grau (Apartat 15.1 del Llibre d'Àlgebra)
Sistemes d'equacions lineals amb dues incògnites (Tema 16 del Llibre
d'Àlgebra)

La funció de primer grau apareix també quan estudiem...


Les rectes en el pla (Tema 2 del Llibre de Geometria Analítica)

Trobareu Problem-solving amb funcions de primer grau al Tema 8 del llibre de


problemes d’àlgebra: http://www.toomates.net/biblioteca/ProblemasAlgebra.pdf

Anomenem funcions lineals a les funcions polinòmiques de primer grau, és a dir,


les funcions de la forma
y  f ( x)  ax  b

on a i b són nombres reals.

La seva gràfica sempre és una recta amb pendent a i que talla l'eix Y al punt
( 0,b )
2.1.1
Determina si les següents funcions són o no funcions de primer grau. Si la funció
és de primer grau, expressa aquesta funció de la forma y  f ( x)  ax  b .

a) f ( x)  2  3x b) f ( x)  31  2 x 
1  5x
c) h( x)  x 4  3x  d) k ( x) 
4

2.1.2
Determina si les següents funcions són o no funcions de primer grau. Si la funció
és de primer grau, expressa aquesta funció de la forma y  f ( x)  ax  b .

c) f ( x)  x4  x 
1
a) f ( x)  2  x b) f ( x)  1  4 x
3
x 1 2x  3
d) f ( x)  x  1 e) f ( x)  f) f ( x) 
5 x
g) f ( x)  3x  1
1
2

Interpretació gràfica del pendent d’una funció lineal i del valor de b

Si y  f ( x)  ax  b , el nombre a s’anomena pendent de la funció.

El nombre b és el punt de tall de la gràfica de la funció amb l’eix Y:


2.1.3
Donades les següents gràfiques,
a) indica el signe i ordena en ordre decreixent els pendents m1 , m2 , m3 i m4
b) indica el signe i ordena en ordre decreixent els valors b1 , b2 , b3 i b4

Determinació del pendent d’una funció lineal que passa per dos punts.

El pendent m1 d’una funció lineal y  f ( x)  ax  b es pot determinar amb dos


punts P1  ( x1 , y1 ) , P2  ( x2 , y 2 ) qualsevol de la seva gràfica:
Exemples.
a) b)

c) d)

e) f)
2.1.4
Determina els pendents a1 , a 2 , a3 , a4 de les següents funcions:
2.1.5
Observant la forma de cada gràfica, determina la seva funció associada:

a) b)

c) d)

2.1.6
Determina les següents funcions lineals y  f ( x)  ax  b

a) Pendent 3, punt de tall Eix Y: -2.


b) Pendent 2/5, punt de tall Eix Y: 4
c) Pendent 5, passa per (2,3) .
d) Pendent -1, passa per (1,7) .
2.1.7
Observant la forma de cada gràfica, determina la seva funció associada:
a) b)

c) d)

e) f)
g) h)

i) j)

2.1.8
Determina l’equació associada a cada gràfica:
2.2 La gràfica de les funcions de primer grau.

Exercici resolt.
Representa gràficament la funció f ( x)  3x  7 .

Solució:
Fem una taula de valors X-Y i els representem en un sistema de coordenades:

Les gràfiques de les funcions de primer grau sempre son rectes, i per tant només
cal fer una taula X-Y amb dos valors

Exemple resolt.
3
Representa gràficament la funció y  f ( x)  x 1
2
3
x  0  y  f (0)  0  1  1
2
3
x  2  y  f (2)  2  1  4
2

Representem gràficament els punts (0,1) i (2,4) els unim amb una recta:
2.2.1
Representa gràficament les següents funcions:

a) f ( x)  2 x  3 b) g ( x)  2 x  1 c) h( x)  0.5x  2
2.2.3
Representa gràficament les següents funcions.
4 5
a) f ( x)  x  2 b) f ( x)  x  2
5 4

7 5
c) f ( x)  x4 d) f ( x)  x5
4 2

1
e) f ( x)  x4 f) f ( x)   x  4
4
2.3 Determinació de funcions de primer grau.

Primer mètode. Determinant el pendent de la recta.

Exemple resolt.
Determina la funció lineal que passa pels punts A  ( 3,1) i B  ( 8 , 4 )

Solució:

La nostra funció serà de la forma y  f ( x)  a x  b , on a és el pendent de la


recta.

x  x2  x1  8  3  5  y 3
a 
y  y2  y1  4  1  3 x 5

3
Per tant, la nostra funció serà de la forma y  f ( x) 
xb
5
El valor de b el podem obtenir substituint en qualsevol dels dos punts:

3 9 9 4
La funció passa pel punt A  1  f (3)  3  b   b  b  1  
5 5 5 5

3 4
La funció és f ( x)  x
5 5

Comprovem el resultat:
3 4 9 4
(3,1)  f (3)   3     1
5 5 5 5
3 4 24 4 20
(8,4)  f (8)   8     4
5 5 5 5 5
Segon mètode. Mitjançant un sistema d'equacions 22.

Exemple resolt.
Determina la funció de primer grau que passa pels punts ( 1, 2 ) i ( 6 , 4).

Solució:
La funció serà de la forma f ( x)  ax  b per a certs a i b que hem de trobar.

Substituïm els valors X-Y de l'enunciat:

(1,2)  2  a  1  b  2  a  b
(6,4)  4  a  6  b  4  6a  b

Resolem el sistema d'equacions que hem obtingut:

2  a  b b  2  a 2
   2  a  4  6a  2  5a  a 
4  6a  b b  4  6a 5
2 8
b  2a  2 
5 5

2 8
La funció és f ( x)  x
5 5

Comprovem el resultat:
2 8 10
(1,2)  f (1)    2
5 5 5
2 8 12 8 20
(6,4)  f (6)  6     4
5 5 5 5 5
2.3.1
Completa la següent taula:

Punt tall eix Punt de la Punt de la


Fórmula Pendent
Y gràfica gràfica
f ( x)  2 x  1
 3, 4   1 , 7 
-2 1
1/2  0 ,1 
1
0
 3,3   3, 2 
 5, 9 
2.4 Punts d’intersecció amb els eixos.

Exemple resolt.
a) Determina la funció lineal que passa pels punts  3 ,1  i  5 , 4  .
b) Deterimina el seu punt de tall amb l’eix Y.
c) Determina el seu punt de tall amb l’eix X.

Solució:
a) La funció és de la forma f ( x)  a x  b . Substituïm en els punts indicats:
 3 ,1   1  a  3  b
 5, 4   4  a  5  b
Resolem el sistema 2 2 obtingut:
1  3a  b b  1  3a 3
   1  3a  4  5a  1  4  5a  3a  3  2a  a 
4  5a  b b  4  5a 2

3 7
b  1  3a  1  3 
2 2
3 7
La funció buscada és f ( x)  x 
2 2

3 7 7
b) Avaluem la funció en x  0 : f (0)  0    3.5
2 2 2
c) Hem de resoldre l’equació f ( x)  0
3 7 3 7 7
f ( x)  0  f ( x)  x   0  x   x   2.33
2 2 2 2 3
2.4.1
Determina la funció lineal que passa pels dos punts indicats. Determina els seus
punts de tall amb els eixos.

a)  2 , 2  ,  5 , 3 

b)  1, 3  ,  4 ,1 
c)  2 ,1  ,  7 , 2 

d)  1,  1  ,  6 , 2 
2.5 Intersecció entre dues funcions de primer grau.

Trobar els punts de tall entre dues gràfiques f (x) i g (x) equival a resoldre
l'equació

f ( x )  g ( x)

Exercici resolt.
Determina els punts d'intersecció de les gràfiques associades a les funcions
f ( x )  x  3 i g ( x)  2 x  4

Solució:
f ( x)  g ( x) 
x  3  2x  4 
3  4  2x  x 
1  x

Busquem punts, per tant, hem de trobar també la "y":


x  1  f (1)  1  3  2

El punt de tall és (  1, 2 )

Fem la comprovació:
f (1)  1  3  2
g (1)  2(1)  4  2  4  2  f (1)

2.5.1
A cada apartat, determina el punt de tall entre les dues gràfiques:
a) f ( x)  4 x  3 i g ( x)  5x  1
b) f ( x)  2 x  1 i g ( x)  3x  2
c) f ( x)  2 x  1 i g ( x)  3x  4
d) f ( x)  4 x  3 i g ( x)  5x  2
2.5.2
Determina l’equació associada a cada recta (visualment, amb el mètode de
l’apartat 2.1) i troba el punt d’intersecció.

a)

b)
c)

d)
2.5.3
Determina les següents parelles de funcions lineals, representa-les gràficament i
determina algèbricament el seu punt de tall. Comprova visualment el punt trobat.

a) f (x) que passa per  2 ,1  i  5 , 4  , g (x) que passa per  5 ,1  i  6 ,  1  .


b) f (x) que passa per 1,  2  i  4 , 4  , g (x) que passa per 1, 3  i  7 ,  3  .
c) f (x) que passa per 1,  5  i  6 ,5  , g (x) que passa per  2 , 6  i  5 ,  3  .
d) f (x) que passa per 1,  3  i  7 ,5  , g (x) que passa per   2 , 4  i
 8, 1  .
2.6 Funcions lineals en context.

2.6.1
Hem construït una presa en un riu amb la finalitat de retenir l’aigua i crear un
embassament. L’altura d’aigua segueix la funció

f (t )  4.5t  28

on t son els anys que han passat des de la seva construcció i f (t ) es mesura en
metres.

a) Representa la gràfica de la funció.


b) Quin és el pendent de la funció?
c) Quina va ser l’altura de l’aigua en el moment de la seva construcció?
d) Quina serà l’altura de l’aigua quatre anys després?
e) Quants anys haurem d’esperar perquè l’altura de l’aigua arribi al 64 metres?

2.6.2
Una bomba omple una piscina a raó de 20 litres per minut. Inicialment aquest
piscina contenia 800 litres d’aigua.

a) Determina la funció lineal que representa el volum V d’aigua de la piscina (en


litres) en funció del temps t en minuts.
b) Quants litres contindrà la piscina després d’una hora?
c) Si la piscina és de 5000 litres, quantes hores necessitarà la bomba per omplir-
la?
2.6.3
Maria i Joan van en cotxe a Alacant, a velocitat constant. Maria surt de
Barcelona i Joan surt de Tarragona. Les següents gràfiques indiquen els
quilòmetres recorreguts per tots dos en funció del temps, en hores.

a) Indica la funció f (t ) que representa els quilòmetres recorreguts per Maria en


funció del temps t de viatge, en hores.
b) Indica la funció g (t ) que representa els quilòmetres recorreguts per Joan en
funció del temps t de viatge, en hores.
c) Determina la distància recorreguda per tots dos després de cinc hores de
viatge.
d) Determina el temps necessitat per cadascun per arribar a Alacant, que es troba
a 526 quilòmetres de distància de Barcelona.

2.6.4
Aprofitant l’èxit del nou disc d’un famós grup de música, una botiga ha posat a
la venda samarretes amb el nom del grup. El benefici que obté per la venda de
samarretes es pot expressar per la funció següent:
B(x) = 14x -300

on x és el nombre de samarretes venudes i B(x) n’és el benefici obtingut.

a) Si han venut 42 samarretes, quin és el benefici que han obtingut?


b) I si la botiga ha obtingut un benefici de 680 €, quantes samarretes s’han
venut?
ACM4 2013 #10.1, #10.2
2.6.5
Dues amigues, la Clara i la Paula, han fet una excursió en bicicleta per la
mateixa carretera i en el mateix sentit. Totes dues surten al mateix moment, però
de punts diferents, la Clara del km 0 i la Paula del km 10.
La distància recorreguda per cada una d‘elles segons el temps transcorregut està
representada en el gràfic següent:

a) Al cap de quantes hores la Clara ha atrapat la Paula?


b) Quants quilòmetres ha recorregut la Paula abans de ser atrapada per la Clara?
c) Quina ha estat la velocitat mitjana de la Clara?

ACM4 2013 #8.1, #8.2, #8.3

2.6.6
El cotxe de la teva família necessita canviar els quatre pneumàtics.
Heu trobat dues botigues que tenen les ofertes següents:

a) Si escolliu la botiga A, quant haureu de pagar pels 4 pneumàtics?


b) Si compreu els 4 pneumàtics a la botiga B, quin és, de mitjana, el preu de
cada pneumàtic?
c) Si el nombre de pneumàtics que voleu comprar és x i el preu total dels
pneumàtics és y, quina és l’expressió algebraica que correspon a la botiga A?
y = 60,30x + 50
y = 60,30x
y = 50x + 95
d) Utilitza la taula següent per calcular el nombre de pneumàtics amb què
comença a ser més cara l’oferta de la botiga A que la de la botiga B.

ACM4 2013 #7.1, #7.2, #7.3, #7.4

2.6.7
Dues ciutats s’han proposat incrementar el tant per cent de la població que fa
una recollida selectiva de residus (orgànica, paper, vidre...).
Han previst que el percentatge de població que fa recollida selectiva
evolucionarà fins a l’any 2018 segons aquest gràfic:

a) L’any 2014, quin percentatge de la població de la ciutat A està previst que


faci la recollida selectiva?
b) A quin any les dues ciutats tindran el mateix percentatge de població que fa
recollida selectiva?
c) Si les previsions s’acompleixen, a quin any està previst que la ciutat B
aconsegueixi que tota la recollida sigui selectiva?
ACM4 2013 #9.1, #9.2, #9.3

2.6.8
En Ferran necessita comprar 4 cartutxos de tinta per a la seva impressor(A) Pot
triar entre dues ofertes.

a) Si escull els 4 cartutxos de l’oferta fixa, quant ha de pagar en Ferran?


b) Si escull els 4 cartutxos de l’oferta variable, quant ha de pagar en Ferran?
c) Utilitza la taula següent per calcular el nombre de cartutxos a partir del qual
comencen a ser més cars els de l’oferta fixa que els de l’oferta variable.

d) Si x és el nombre de cartutxos que es compren i y és el preu total dels


cartutxos, marca amb una X l’expressió algebraica corresponent al preu de
l’oferta fixa.
y = 10x + 8
y = 17x +8
y = 10x

ACM4 2013 #8.1, #8.2, #8.3, #8.4

2.6.9
El benefici B(x) que obté el propietari del web depèn del nombre de
descàrregues fetes i respon a l’expressió següent:
B(x) = 4x – 600

on x és el nombre de descàrregues fetes i B(x) és el benefici obtingut en E.


a) Si s’han fet 220 descàrregues, quants euros ha tingut de benefici?
b) Si el propietari del web ha obtingut un benefici de 1.000 €, quantes
descàrregues s’han fet?

ACM4 2013 #6.1, #6.2

2.6.10
Una família vol anar amb cotxe a Norueg(A) Pensen arribar al Cap Nord.
De Barcelona a Oslo, capital de Noruega, faran 2.400 km i des d’Oslo al Cap
Nord faran 2.250 km.

a) Suposant que fins a Oslo fan uns 600 km diaris de mitjana i que després fins
al Cap Nord fan uns 225 km diaris, quants dies han de preveure per fer el viatge
d’anada de Barcelona al Cap Nord?
b) Des d’Oslo al Cap Nord la família suposa que el cotxe consumirà 8 litres de
gasolina cada 100 km (8 litres/100 km). Quants litres de gasolina consumirà el
cotxe en aquesta part del viatge (2.250 km)?
c) Abans de marxar, diversos membres de la família fan proves de conducció. La
Natàlia condueix un dia durant tres hores. La primera hora viatja a 100 km/h; la
segona, a 25 km/h, i la tercera, a 50 km/h. Quin gràfic representa millor la prova
de conducció realitzada per la Natàlia?

ACM4 2014 #4.1, #4.2, #4.3

2.6.11
Hi ha moltes botigues a Internet que venen música. En Roger ha trobat dues
ofertes per descarregar-se cançons:

OFERTA A: 12 euros d’inscripció i 0,50 euros per cançó


OFERTA B: 1,10 euros per cançó

Observa aquest gràfic que representa les dues ofertes per descarregar cançons.

En Roger vol descarregar-se cançons. Quina és l’oferta més barata depenent del
nombre de cançons descarregades? Justifica la resposta.
ACM4 2014 #2.6
2.6.12
S’han utilitzat escuradents per dibuixar aquestes figures.

a) Quants escuradents es necessitarien per fer la figura 5?


(A) 5 (B) 19 (C) 23 (D) 25

b) Quina de les afirmacions següents és correcta?


(A) La diferència del nombre d’escuradents entre dues figures consecutives és 4.
(B) El nombre d’escuradents d’una figura sempre és múltiple de 3.
(C) Les figures sempre tenen un nombre parell d’escuradents.
(D) El nombre d’escuradents de cadascuna de les tres primeres figures és un
nombre primer.

c) El nombre d’escuradents necessaris per fer una figura segons el seu número
s’observa a la taula següent:

d) Quina és l’expressió algebraica (y) corresponent al nombre d’escuradents


de la figura n?
(A) y = 3n + 4 (B) y = 4n + 3 (C) y = 4n + 7 (D) y = 7n + 4

ACM4 2015 #11, #12, #13

2.6.13
Observa el gràfic de l’evolució d’ingressos i de despeses d’una empresa des de
l’any 2012.

a) L’any 2013, els beneficis han estat de... (Recorda: beneficis = ingressos –
despeses)
(A) 1 milió d’euros. (B) 2 milions d’euros. (C) 4 milions d’euros. (D) 5
milions d’euros.

b) Quin any els beneficis han estat de 3 milions d’euros?


(A) 2012 (B) 2013 (C) 2014 (D) 2015

c) Justifica l’afirmació següent: “Cada any tenim 1 milió d’euros de beneficis


més que l’any anterior”.

ACM4 2016 #27, #28, #29

2.6.14
Aquest gràfic mostra el preu total (y) d’una broca més una quantitat (x) de
paquets de tacs especials.

Segons aquest gràfic, quin és el preu d’una broca i el d’un paquet de tacs
especials? Fes els càlculs i el raonament aquí:

ACM4 2017 #27


2.6.15
L’Anna, l’Albert i l’Olga formen un equip per participar en un concurs de
llançaments d’avions de paper.
Per inscriure’s al concurs, s’ha de pagar una quota fixa d’inscripció, més una
quantitat que varia segons el nombre de membres de l’equip. El gràfic següent
mostra l’evolució del preu d’inscripció d’un equip segons el nombre de membres
que el componen. Quant costa la quota fixa d’inscripció i quant es paga per cada
membre de l’equip?
Explica clarament com obtens els resultats.

ACM4 2018 #26


2.6.16
Quan els estudis l’hi permeten, la Lorena ajuda els seus pares a la papereria, en
l’enquadernació artesanal de llibretes.

La Lorena determina que el preu d’una llibreta, l’any 2017, el dona l’expressió
següent:

y = 10x + 50,

on y és el preu en cèntims d’euro de la llibreta i x és la quantitat de fulls de la


llibret(A) a) Si una llibreta té 30 fulls, quin és el preu d’aquesta llibreta?

(A) 2,50 euros (B) 3 euros (C) 3,50 euros (D) 4 euros

b) I si una llibreta costa 2 euros, quants fulls té? (Recorda que y = 10x + 50, on y
és el preu en cèntims d’euro de la llibreta i x és la quantitat de fulls utilitzats)

(A) 10 fulls (B) 15 fulls (C) 20 fulls (D) 60 fulls

ACM4 2018 #8, #9

2.6.17
En acabar el curs, els alumnes de 4t d’ESO fan una sortida a un parc aquàtic. El
parc facilita el material per fer les diverses activitats que s’ofereixen. El preu del
lloguer del material té dues parts: una part fixa i una part variable de 0,40 euros
per minut, segons els minuts d’ús d’aquest material.
a) Avui, el parc aquàtic fa l’oferta següent: els primers 10 minuts no es cobra la
part variable del lloguer. A l’expressió següent:
y = 3 + 0,40 · (x - 10) ,

la x representa el temps d’ús del material en minuts, i la y representa el preu del


lloguer del material en euros. Quin preu té la part fixa del lloguer de material?
(A) 3 euros (B) 5 euros (C) 17 euros (D) 23 euros

b) Si un alumne vol estar una hora a l’activitat, quant li costarà el lloguer del
material?
(A partir de l’expressió y = 3 + 0,40 · (x - 10))
(A) 3 euros (B) 7 euros (C) 23 euros (D) 27 euros
c) Un altre alumne ha gastat 19 euros en el lloguer del material per a l’activitat.
Quant de temps hi ha estat? (a partir de l’expressió y = 3 + 0,40 · (x - 10))
(A) 19 minuts (B) 40 minuts (C) 50 minuts (D) 55 minuts

ACM4 2019 #14, #15, #16

2.6.18
L’Anna és una venedora que té un sou mensual format per una part fixa (600
euros) i una part variable, segons el nombre de vendes que hagi fet. Així, el seu
sou s’expressa per:
S(x) = 30x + 600

On x és el nombre de vendes fetes i S(x) és el sou mensual en euros.

a) Si ha fet 30 vendes en aquest mes, quin ha estat el sou de l’Anna?


(A) 630 euros (B) 900 euros (C) 1.500 euros (D) 18.900 euros

b) Si el mes passat l’Anna va cobrar 1.800 euros, quantes vendes va fer?


(A) 30 vendes (B) 40 vendes (C) 60 vendes (D) 630 vendes

ACM4 2020 #30, #31

2.6.19
En Marc vol fer 16 fotocòpies i en una botiga de reprografia s’ofereixen les dues
opcions següents:
OPCIÓ A: Cobra 10 cèntims d’euro per cada fotocòpia.
OPCIÓ B: Cobra 30 cèntims fixos per la primera fotocòpia i 7 cèntims d’euro
per cada fotocòpia addicional.

a) Si escull l’opció A, quants euros haurà de pagar en Marc per les 16


fotocòpies?
(A) 1,10 euros (B) 1,30 euros (C) 1,60 euros (D) 1,65 euros

b) Si escull l’opció B, quants euros haurà de pagar en Marc per les 16


fotocòpies?
(A) 1,30 euros (B) 1,35 euros (C) 1,80 euros (D) 2,10 euros

c) Utilitza la taula següent per calcular el nombre de fotocòpies a partir del qual
comença a ser més cara l’opció A que la B.

(A) 6 fotocòpies (B) 7 fotocòpies (C) 8 fotocòpies (D) 10 fotocòpies


d) Si x = nombre de fotocòpies que es fan. y = cost total, en cèntims, de les
fotocòpies fetes. Quina és l’expressió algebraica que permet calcular el cost
total, en cèntims, de les fotocòpies si s’escull l’opció A?
(A) y = 16x (B) y = 10x (C) y = 16x + 10 (D) y = 10x + 16

ACM4 2020 #23, #24, #25, #26

2.6.20
Per protegir dos tipus de peixos A i B, es va acordar que, durant uns anys, calia
reduir-ne les captures. El nombre màxim de tones de captures permeses de
peixos durant aquests anys es va recollir en el gràfic següent:

a) Segons el gràfic, quin any van estar permeses les mateixes tones de captura
per als dos tipus de peix?
(A) 2016 (B) 2017 (C) 2018 (D) 2019

b) Si es manté la mateixa tendència, quantes tones de captures estarien permeses


per als peixos de tipus A, l’any 2020?

(A) 1.000 tones (B) 2.000 tones (C) 3.000 tones (D) 4.000 tones

c) Quin és, en tones, el decreixement anual de captures permeses dels peixos de


tipus B?

(A) 1.000 tones (B) 2.000 tones (C) 4.000 tones (D) 6.000 tones

ACM4 2020 #1, #2, #3

2.6.21
L’Eva envia les comandes mitjançant una empresa de missatgeria. Per cada
enviament cal pagar una quantitat fixa més una quantitat que varia segons el pes
del paquet. Observa el gràfic. Quina és la quantitat fixa que cal pagar per cada
enviament i quant cal pagar exactament per cada quilogram del paquet?
ACM4 2021 #11
3 Les funcions de segon grau.

Ara és un bon moment per repassar...


Les equacions de segon grau. (Tema 11 del Llibre d'Àlgebra)

Les funcions de segon grau apareixen també quan estudiem...


Les paràboles (Tema 4 del Llibre de Geometria)

3.1 Funcions de segon grau en forma general.


Una funció es diu que és polinòmica de segon grau, o funció quadràtica, si és
del tipus:

y  f ( x)  a x 2  b x  c

on a, b, i c són nombres qualssevol, i a mai és zero.

La gràfica de la funció de segon grau és sempre una paràbola.

Punt de tall amb l’eix Y.


Avaluem la funció per a x  0  f (0)  a 0 2  b  0  c  c

Punts de tall amb l’eix X.


Són les (possibles) solucions de l’equació f ( x)  0  a x 2  b x  c  0

Vèrtex de la paràbola.
b  b b 
Anomenem vèrtex de la paràbola, al punt x    , f  
2a  2a  2a  

El vèrtex d’una pàràbola és el punt mínim (si a  0 ) o un màxim (si a  0 ).

Eix de simetria de la paràbola.


Anomenem eix de simetria de la paràbola a la recta (vertical) que al “doblegar”
el full faria que els dues “branques” coincidissin.
b
L’eix de simetria, és la recta vertical d’equació: x 
2a
Important: Per trobar la gràfica d’una funció quadràtica s’ha de buscar, en
primer lloc, el vèrtex d’aquesta gràfica. A continuació, s’han de buscar parells de
punts equidistants del vèrtex; com més parells de punts es trobin, més precisa
serà la representació de la paràbola. A més, en tota paràbola és convenient
assenyalar els punts de talls amb els eixos.
3.1.1
Les funcions de segon grau x2 i –x2.

Representa gràficament les funcions y  f ( x)  x 2 i y  g ( x)   x 2

x y  f ( x)  x 2
-4 y  f (4)   4  16
2

-3
-2
-1
0
1
2
3
4
x y  g ( x)   x 2
-4 y  g (4)  4 2  16
-3
-2
-1
0
1
2
3
4

Completa la següent taula :

Funció y  f ( x)  x 2 y  g ( x)   x 2
Domini
Recorregut
Punts de tall amb l’eix X
Punts de tall amb l’eix Y
Simetria
Creixement i decreixement
Màxims i mínims
Curvatura
Exercici resolt
Representa gràficament la funció y  f ( x)  x 2  2

Solució:
Fem una taula de valors X-Y:

i els passem a un sistema de coordenades:


3.1.2
Representa gràficament la funció y  f ( x)  x 2  x  6

x y  f ( x)  x 2  x  6
-4 y  f (4)  (4) 2  (4)  6  14
-3
-2
-1
0
1
2
3
4

Completa la següent taula :

y  f ( x)  x 2  x  6
Funció

Domini
Recorregut
Punts de tall amb l’eix X
Punts de tall amb l’eix Y
Simetria
Creixement i decreixement
Màxims i mínims
Curvatura
3.1.3
Representa gràficament la funció y  f ( x)  x 2  x  2

x y  f ( x)  x 2  x  2
-4 y  f (4)  (4) 2  (4)  2  18
-3
-2
-1
0
1
2
3
4

Completa la següent taula :

y  f ( x)  x 2  x  6
Funció

Domini
Recorregut
Punts de tall amb l’eix X
Punts de tall amb l’eix Y
Simetria
Creixement i decreixement
Màxims i mínims
Curvatura
3.1.4
Punts notables de la gràfica de segon grau.
a) Donada la funció f ( x)  x 2  4 x  1 . Determina:

- Punt de tall amb l’eix Y.


- Punts de tall amb l’eix X.
- Vèrtex.
- Eix de simetria.

Representa els elements anteriors en un sistema X-Y i amb la seva ajuda traça la
gràfica de la funció.

Fes ara el mateix amb les següents funcions:


b) f ( x)  2 x 2  4 x  3
c) f ( x)   x 2  5x  1
d) f ( x)  x 2  3x  10
Punts de tall amb l'eix X.
Determinar els punts de tall amb l'eix X equival a resoldre una equació de segon
grau. Ho hem d'intentar per factorització, i hem d'intentar factoritzar la funció
mentalment. Si no, aplicarem la fórmula.

Exercici resolt.
Determina els punts de tall amb l'eix X de la funció f ( x)   x 2  5x .

Solució:
Hem de resoldre l'equació  x 2  5x  0
0   x 2  5x  x x  5 Traiem factor comú x
x  0
0  x x  5   Apliquem el Principi del producte
 x  5  0  x  5
nul.

Els punts de tall són x  0 i x  5


3.2 Funcions de segon en forma estàndard.

Tota funció de segon grau es pot escriure de la forma

f ( x)  a( x  h)2  k

I en aquest cas, l'eix de simetria és la recta vertical x  h i el vèrtex de la funció


és el punt (h, k ) .

Si a  0 la gràfica estarà oberta cap amunt, i si a  0 , la gràfica estarà oberta


cap avall.

La gràfica de la funció f ( x)  a( x  h)2  k és la gràfica de la funció


g ( x)  ax 2 , sotmesa a una translació horitzontal h unitats a la dreta i una
translació vertical de k unitats.

Exemple:
Representa gràficament la funció f ( x)  2( x  3)2  8 .

Partim de la gràfica de la funció g ( x)  2 x 2 , que té les branques cap avall:

Punts de tall amb els eixos.


Si la funció ve expressada en forma estàndard, els punts de tall amb l'eix X es
poden determinar sense necessitat de resoldre cap equació de segon grau.

Per exemple, amb la funció anterior f ( x)  2( x  3)2  8

f ( x)  2( x  3) 2  8  0  2( x  3) 2  8 
8 x  2  2  x  4
( x  3) 2   4  x  3   4  2  
2  x  3  2  x  1
3.2.1
Donada la funció
f ( x)  ( x  3)2  1

Determina el seu eix de simetria, el vèrtex, els punts de tall amb els eixos i
representa-la gràficament.

3.2.2
Associa cada funció amb la seva corresponent gràfica.

f ( x)  ( x  1)2  1 g ( x)  ( x  1)2  1 h( x)  ( x  1)2  1


j ( x)  ( x  1)2  1

3.2.3
Determina les coordenades del vèrtex de la gràfica de cada funció:
a) f ( x)  2( x  3)2  1 b) f ( x)  3( x  2)2  12
c) f ( x)  2( x  1)2  5 d) f ( x)  2( x  4)2  8
e) f ( x)  2 x 2  8x  3 f) f ( x)  3x 2  12 x  1
g) f ( x)   x 2  2 x  8 h) f ( x)  2 x 2  8x  1
Com passar de forma general a forma standard (amb a=1).

Exemple resolt 1.
Escriu en forma standard la funció f ( x)  x2  6 x  7

Solució:
f ( x)  x 2  6 x  7  x 2  2  3x  7  x 2  2  3x  3 2  3 2  7 
 x  3  32  7  x  3  2
2 2

Exemple resolt 2.
Escriu en forma standard la funció f ( x)  x2  10 x  15

Solució:
f ( x)  x 2  10 x  15  x 2  2  5 x  15  x 2  2  5 x  5 2  5 2  15 
( x  5) 2  5 2  15  ( x  5) 2  10

3.2.4
Determina la forma standard de les següents funcions. Després, i sempre sobre la
base de la forma standard de la funció:
a) Interpreta i representa la seva gràfica com a composició de translacions de la
funció base x 2 .
b) Determina el vèrtex.
c) Determina els seus punts de tall amb els eixos.

a) f ( x)  x2  8x  4 b) f ( x)  x2  6 x  5 c) f ( x)  x2  14 x  11
d) f ( x)  x2  6 x  17 e) f ( x)  x2  14 x  27 f) f ( x)  x2  10 x  9
g) f ( x)  x2  5x  1 h) f ( x)  x2  x  7 i) f ( x)  x2  3x  7
j) f ( x)  x 2  11x  30

3.2.5
A cadascuna de les funcions següents, i sense representar-la gràficament,
dedueix si té un màxim o un mínim i quines són les seves coordenades:
a) f ( x)  3x 2  12 x  1 b) f ( x)  2 x 2  8x  3 c) f ( x)  4 x 2  8x  3
d) f ( x)  2 x 2  12 x  3 e) f ( x)  5x 2  5x f) f ( x)  6 x 2  6 x

3.2.6
a) Escriu la seva forma estàndard.
b) Determina el seu vèrtex V, els punts de tall amb l'eix X i el punt de tall amb
l'eix Y.
c) Dibuixa la seva gràfica.
d) Determina el domini i el recorregut.

a) f ( x)  x 2  2 x  3 b) f ( x)  x 2  6 x c) f ( x)  3x 2  6 x
d) f ( x)   x 2  6 x  4 e) f ( x)  2 x 2  4 x  3 f)
f ( x)  4 x 2  12 x  1
3.2.7
Determina la funció de segon grau coneguts el seu vèrtex i un punt de la seva
gràfica. Dóna el resultat en forma estàndard.

a) Vèrtex ( 0 , 0 ) i passa pel punt ( -2 , 8 ).


b) Vèrtex ( 2 , 0 ) i passa pel punt (1 , 3).
c) Vèrtex ( -3 , 0 ) i passa pel punt ( -5 , -4 ).
d) Vèrtex ( 0 , 1 ) i passa pel punt ( -1 , 0 ).
e) Vèrtex ( 2 , 5 ) i passa pel punt ( 3 , 7 ).
f) Vèrtex ( -3 , 4 ) i passa pel punt ( 0 , 0 ).

3.2.8
Passa cadascuna de les següents funcions a forma estàndard aplicant el mètode
de completar quadrats. Un cop passada a forma estàndard, determina el seu
vèrtex.

a) f ( x)  x 2  4 x b) f ( x)  x 2  2 x  2 c) f ( x)  6 x  10  x 2
d) f ( x)  8  3x  x 2 e) f ( x)  2 x 2  8x  9

3.2.9 Exercici del Youtube


Determina la forma standard i amb ella determina les coordenades del vèrtex de
les següents funcions de segon grau:

a) f ( x)  x 2  12 x  15
Solució: https://youtu.be/Rwsb5vrMJ1c

b) f ( x)   x 2  8x  9
Solució: https://youtu.be/YD7R2vzrngA

c) f ( x)  5x 2  30 x  11
Solució: https://youtu.be/Ud2YRjs5NXY
3.3 Determinació de funcions de segon grau (amb un punt a l’eix Y).

Exemple resolt.
Determina la funció de segon grau que passsa pels punts

A   0 ,  40  , B    3,  16  , C   6 , 20 

Solució:
La funció que busquem és de la forma f ( x)  a x 2  b x  c

A   0 ,  40   40  f (0)  a 02  b 0  c  c  c  40


B    3,  16   16  f (3)  a (3) 2  b (3)  c  16  9a  3b  40

C   6 , 20   20  f (6)  a 62  b 6  c  20  36a  6b  40

Resolem el sistema 2x2 :


 24  9a
 b
 16  9a  3b  40 24  9a  3b  3
   
 20  36 a  6b  40 60  36 a  6b b   36a
60
 6
24  9a 60  36a 24  9  2
 a 2b  2
3 6 3

La funció buscada és f ( x)  2 x 2  2 x  40 .

Comprovació:
f (0)  2  02  2  0  40  40
f (3)  2  (3) 2  2  (3)  40  16
f (6)  2  6 2  2  6  40  20
3.3.1
Determina la funció de segon grau que passa pels punts indicats. Determina els
seus punts notables (punt d’intersecció amb l’eix Y, punts d’intersecció amb
l’eix X, vèrtex, eix de simetria)

a) A   0 ,  8  , B  1,  9  , C    3, 7 
b) A   0 ,  15  , B   3,  12  , C    2 ,  7 
c) A   0 ,10  , B   2 ,  6  , C    3, 64 
d) A   0 ,15  , B  1,16  , C    2 , 7 
e) A   0 , 6  , B   4 ,  30  , C    2 ,  12 

3.3.2
a) Determina la paràbola que passa pels punts  0 , 4  ,   1, 5  i  1,1  , com a
funció de segon grau.

b) Determina els seus punts de tall amb els eixos. Comprova que els resultats
obtinguts son coherents amb el gràfic.
3.3.3 Three-Act Math

Funcions de segon grau en context: “will it hit the hoop” (adaptat)

a) Vídeo presentació: Multimedia\will it hit the hoop\act1.mov

Dades: A   0 , 6  , B   5 ,11.75  , C   10 ,14  , Objectiu: P   18 ,10.3 

b) Vídeo presentació: Multimedia\will it hit the hoop\sequels\take2\act1.mov

Dades: A   0 , 5  , B   5 ,11.5  , C   10 ,14  , Objectiu: P   18 ,10.3 


3.4 Determinació de funcions de segon grau (cas general).

Ja vam veure a l'apartat anterior que tota funció de primer grau f ( x)  ax  b


queda determinada per dos punts de la seva gràfica, i que necessitem resoldre un
sistema de dues equacions i dues incògnites.

Una funció de segon grau f ( x)  ax 2  bx  c queda determinada per tres punts


de la seva gràfica. La seva determinació ens obliga a resoldre un sistema de tres
equacions amb tres incògnites, però veurem amb el següent exemple que és més
fàcil del que sembla, que té truco.

Exercici resolt.
Determina la funció de segon grau f ( x)  ax 2  bx  c sabent que
f (2)  5 , f (1)  8 , f (6)  4 , és a dir, que passa pels punts (2,5) , (1,8) i
(6,4) .

Solució:

Substituïm els valors donats a l'expressió algebraica:


f (2)  5  5  f (2)  a  (2)2  b  (2)  c  4a  2b  c
f (1)  8  8  f (1)  a  12  b  1  c  a  b  c
f (6)  4  4  f (6)  a  62  b  6  c  36a  6b  c
Per tant, hem obtingut el següent sistema de tres equacions i tres incògnites:
5  4a  2b  c

8  a  b  c
4  36a  6b  c

Però, observa que la part "  c " desapareix! Per exemple, restant la segona
equació a la primera:
5  4a  2b  c
  3  3a  3b
8  a  b  c

restant la tercera equació a la segona:


8  a  b  c
  4  35a  5b
4  36a  6b  c
i obtenim un sistema de dues equacions amb dues incògnites:
 3  3a  3b

 4  35a  5b

Que podem resoldre, per exemple, per substitució:


 3  3a  3b  1  a  b  a  1  b

 4  35a  5b

31
 4  35(1  b)  5b  35  35b  5b  4  35  35b  5b  31  40b  b 
40

31  40 31  9
a  1  b  1    
40 40 40 40

Un cop resolt el sistema de dues equacions, determinem la "c" amb qualsevol de


les tres equacions inicials:

  9   31 
5  4a  2b  c  5  4   2   c 
 40   40 
4  9 2  31 5  40  4  9  2  31 298 149
c 5    
40 40 40 40 20

Així doncs, la nostra funció és


9 2 31 149
f ( x)   x  x
40 40 20

Comprovem que ho hem fet bé:


9 31 149
f (2)   (2) 2  (2)  5
40 40 20
9 31 149
f (1)   12  1  8
40 40 20
9 31 149
f (6)   6 2  6  4
40 40 20

3.4.1
A cada apartat, determina la funció de segon grau que passa pels següents punts:
a) ( 1, 6 ) , ( 3 , 20 ) , ( -2 , 15 ) b) ( 2, 7 ) , ( -1 , -2 ) , ( 4 , 3)
c) ( 1, -1 ) , ( -1 , 9 ) , ( 3 , 13)
3.5 Punts de tall amb funcions de segon grau.

Trobar els punts de tall entre dues gràfiques f (x) i g (x) equival a resoldre
l'equació
f ( x )  g ( x)

Primer grau vs. Segon grau. Exercici resolt.


Determina els punts d'intersecció de les gràfiques associades a les funcions
f ( x )  x 2  6 x  5 i g ( x)  x  5

Solució:
f ( x)  g ( x)  x 2  6 x  5  x  5  x2  6 x  5  x  5  0  x 2  7 x  10  0

(Recorda: sempre hem d'intentar factoritzar i no tirar de fórmula i sempre


intentem factoritzar de cap)

Busquem dos nombres que el seu producte sigui 10 i la seva suma -7... provant...
provant... Ja està! -5 i -2

x  5  0  x  5
Per tant x 2  7 x  10  ( x  5)( x  2)  0  
x  2  0  x  2

Busquem punts, per tant, hem de trobar també les "y":


x  5  g (5)  5  5  0
x  2  g (2)  2  5  3

Comprovem que, efectivament, són punts de tall:


x  5  f (5)  52  6  5  5  25  30  5  0  g (5)
x  2  f (2)  22  6  2  5  4  12  5  3  g (2)

Els punts de tall són (5,0) i (2,3)


Exemple resolt.
Determina els punts de tall entre les gràfiques f ( x)  x  6 i g ( x)  x 2  4 x

Solució:
f ( x)  g ( x) 
x  6  x 2  4x 
0  x 2  4x  x  6 
 x  1
0  x 2  5 x  6  ( x  1)( x  6)  
x  6

3.5.1
Determina els punts d'intersecció de les gràfiques associades a les funcions
a) f ( x)  x 2  4 x  5 i g ( x)  3x  1
b) f ( x)   x 2  2 x  3 i g ( x)  3x  7
c) f ( x)  x 2  4 x  5 i g ( x)  2 x  3

(Recorda: Resol les equacions factoritzant, sense fer servir la fórmula"menys b


més menys arrel...")
Gràfiques tangents.
Direm que dues gràfiques són tangents quan tenen un únic punt de tall. A aquest
punt de tall únic li direm "punt de tangència".

3.5.2
Determina els punts d'intersecció de les gràfiques associades a les funcions
f ( x)  x 2  8x  12 i g ( x)  2 x  13

3.5.3
Determina els punts de tall entre f ( x)  x 2  3x  2 i g ( x)  x  2 sense fer
servir la calculadora. Representa gràficament aquestes dues funcions i comprova
que, efectivament, els punts trobats són punts d'intersecció.
3.5.4
Determina els punts d’intersecció entre la paràbola i la recta següents:

a) f ( x)  x 2  3x  5 , g ( x)  x  2

b) f ( x)  2 x 2  3x  1 , g ( x)  x  1.5

x 5
c) f ( x)  x 2  2 x , g ( x) 
3

x 1
d) f ( x)  3x 2  4 x  5 , g ( x)  e) f ( x)  x 2  1 , g ( x)  3x  4
2
f) f ( x)   x 2  5x  3 , g ( x)  3x  4 g)
f ( x)  2 x 2  5 x  7 , g ( x)  3 x  2
3.5.5
Determina els punts de tall entre les rectes i paràboles següents, de forma gràfica
i algèbrica :
a) f ( x)  2 x  3 , g ( x)  x 2  x  12

b) f ( x)   x  1 , g ( x)  x 2  4
x3
c) f ( x)  , g ( x)   x 2  5 x  4
2
d) f ( x)  2 x  7 , g ( x)  2 x 2  14 x  20

3.5.6
a) Determina el punt d’intersecció de la paràbola determinada pels punts
P = ( 0 , -30 ) , Q = ( 3 , -24 ) i R = ( -2 , -14)

i la recta deteminada pels punts S=( 2 ,16 ) i T=( 8 , 31 ).

b) Determina el punt d’intersecció de la paràbola determinada pels punts


P = ( 0 , 28 ) , Q = ( 3 , 28 ) i R = ( -1 , 24 )

i la recta determinada pels punts S=( -5 ,10 ) i T=( 5 , -20 ).


3.5.7
a) Determina la paràbola que passa pels punts ( 0 , 1 ) , (1, -2 ) i ( 5 , 6 ), com a
funció de segon grau.
b) Determina la recta que passa pels punts ( -3 , -4 ) i ( 3 , 2 ), com a funció de
primer grau.
c) Representa gràficament aquestes dues funcions al mateix sistema de
coordenades, assenyalant els seus punts de tall.
d) Determina algèbricament els punts de tall anteriors, i comprova la coherència
amb els punts assenyalats a l’apartat anterior.

3.5.8
Representa les següents parelles de funcions.
Utilitza per a representar la gràfica de la funció de segon grau els seus punts
notables (Punt de tall amb l’eix Y, punts de tall amb l’eix X, vèrtex, eix de
simetria)
Determina de forma algèbrica els seus punts de tall i comprova que els resultats
obtinguts són coherents amb les gràfiques dibuixades.

a) f ( x)  x 2  x  2 , g ( x)  2 x  1 b) f ( x)  x 2  x  2 , g ( x)  x  4
c) f ( x)   x 2  x  7 , g ( x)  x  3 d) f ( x)  x 2  6 x  8 ,
g ( x)  2 x  8
3.5.9 Punts de tall entre dues funcions de segon grau. Exemple resolt.
Determina els punts de tall entre les gràfiques de les funcions
f ( x )  x 2  3 x  2 i g ( x)   x 2  2 x  3

Solució.
Dibuixem la gràfica de f ( x)  x 2  3x  2 :
 b  (3) 3
Vèrtex: x     1.5
2a 2 1 2
Fem una petita taula de valors amb punts a la dreta i a l’esquerra del vèrtex:

x -1 0 1 2 3 4
y  f (x) 2 -2 -4 -4 -2 2

Dibuixem la gràfica de g ( x)   x 2  2 x  3 :
2
Vèrtex: x  1
2(1)
Fem una petita taula de valors amb punts a la dreta i a l’esquerra del vèrtex:

x -2 -1 0 1 2 3 4
y  g (x) -5 0 3 4 3 0 -5

Observem que les dues gràfiques es tallen en dos punts: A l’esquerra, entre -0.5
y -1, i a la dreta, entre 3 i 3.5.
Determinem aquests dos punts de forma algèbrica:
f ( x)  g ( x)  x 2  3 x  2   x 2  2 x  3  2 x 2  5 x  5  0 
 b  b 2  4ac 5  (5) 2  4  2  (5) 5  65
x  
2a 22 4
5  65
x  3.266  y  f ( x)  1.131
4
5  65
x  0.766  y  f ( x)  0.885
4
Observem que els valors obtinguts algèbricament corresponen visualment amb
els punts de tall de les gràfiques representades.
3.5.10
Representa i determina algèbricament els punts de tall entre les següents parelles
de funcions. Comprova visualment que els valors obtinguts corresponen amb els
punts trobats a les gràfiques.

a) f ( x )  x 2  2 x  5 , g ( x)   x 2  4 x  1
b) f ( x)  2 x 2  3 x  1 , g ( x)   x 2  5 x  3
c) f ( x)  x 2  3 x  5 , g ( x)  x 2  2 x  1
d) f ( x)  x 2  5 x  2 , g ( x )  x 2  2 x  3

3.5.11
Determina els punts P i Q d’intersecció entre la recta f (x) i la paràbola g (x) ,
amb la següent informació:

a) A   2 , 5  , B   5 , 3.5  , C    1 , 3  , D   0 ,  3  , E   2 ,  9 

b) A    1 ,  3  , B   2 , 3  , C    1 , 2  , D   0 , 5  , E   2 , 5 
3.6 Desigualtats amb funcions de segon grau.

Exemple resolt.
Resol les següents inequacions de forma gràfica:
a) 2 x 2  3  5x  6 b) 2 x 2  3  5x  6 c) 2 x 2  3  5x  6

Solució.
Interpretem aquest problema comparant les gràfiques de les funcions
f ( x )  2 x 2  3 i g ( x)  5 x  6

a) Trobem els seus punts de tall :


 x  0.5
f ( x)  g ( x)  2 x 2  3  5 x  6  
x  3

b) Volem determinar l’interval en el què f ( x)  g ( x) , observant les seves

gràfiques:
Solució :   0.5 , 3 

c) Volem determinar l’interval en el què f ( x)  g ( x) . El determinant observant


les seves gràfiques :

Solució :    ,  0.5    3 ,   
3.7 Transformació de funcions de segon grau.

3.7.1
Escriu cada funció com a composició de la funció base f ( x)  x 2 , i representa-
les gràficament, (sense fer cap taula de valors, només amb translacions).

a) g ( x)  x 2  3 b) f ( x)  x 2  1 c) f ( x)  x  2
2

d) f ( x)  x  4 e) f ( x)  x  6  2 f) f ( x)  x  9  5
2 2 2

g) f ( x)  x  7  1 h) f ( x)  x  10  3
2 2
3.8 Funcions de segon en forma factoritzada.
La forma factoritzada d’una funció de segon grau és

f ( x)  ax  p x  q 

3.8.1 Problema
Si definim la funció f de la forma f ( x)  cx  6x  1 , amb c  5 . Determina
el valor que és igual a f (7) .

(A) f (78) (B) f (12) (C) f (2) (D) f (78)


3.9 Funcions de segon grau en context.

3.9.1
El benefici B(x), en euros, que obté una empresa per la venda de x unitats d’un
determinat producte es representa per la funció

B( x)   x 2  300 x  16100 per a x  0

a) Calculau el benefici de vendre’n 110 unitats.


b) Representau gràficament la funció.
c) Quantes unitats ha de vendre per tal que el benefici sigui màxim? Quin és
aquest benefici màxim?
d) Quantes unitats ha de vendre per tenir un benefici igual a 3900 euros? I per
tenir un benefici superior a 3900 euros?
e) Per a quin interval d’unitats venudes l’empresa no tindrà pèrdues?

3.9.2
Els beneficis setmanals d’una empresa expressats en euros, quan fabrica i ven x
objectes, s’ajusten a la funció

Bx   0.75x 2  75x  1200 , en que 20  x  80

a) Calcular el benefici que obté en fabricar i vendre 20 objectes.


b) Cercau el nombre d’objectes que ha de fabricar i vendre per obtenir el
benefici màxim, així com aquest benefici màxim.
c) El benefici mitjà per x objectes és M(x) = B(x)/x. Digau quants objectes ha de
fabricar i vendre perquè el benefici mitjà sigui màxim, i quin és aquest benefici.

3.9.3
El rendiment dels treballadors d’una fàbrica (valorat en una escala de 0 a 100),
durant una jornada de 8 hores, ve donat per la funció

 10t 2  60t si 0  t  4

r (t )  80 si 4  t  6
170  15t si 6  t  8

on t és el temps en hores.
a) Determinau els intervals de creixement i decreixement. Quin és el rendiment
màxim?
b) En quins instants de la jornada laboral el rendiment se situa a la meitat de
l’escala?
3.9.4
El benefici en euros de la fabricació de x unitats diàries d’un determinat
producte és

 12 x 2  1512 x  11628

a) Calculau el nombre d’unitats diàries que s’han de fabricar d’aquest producte


per tal d’obtenir un benefici màxim.
b) Quin és aquest benefici màxim?

3.9.5
En una nit i el matí següent, la temperatura T (en graus centígrads) d'una certa
regió
varia amb el temps t segons la funció

T (t )  t 2  9t  8 , 0  t  12

a) Quina temperatura hi havia a les 2 del matí?


b) Quina va ser la temperatura máxima?
c) Quin va ser l'interval de variació de la temperatura des de les 0 hores a les 12
hores
d) A quina hora hi va haver una temperatura de zero graus?
4 Les funcions polinòmiques de tercer grau i superior.

Ara és un bon moment per repassar...


Polinomis i operacions amb polinomis. (Tema 9 del Llibre d'Àlgebra)
Factorització i equacions polinòmiques. (Tema 10 del Llibre d'Àlgebra)

4.1 Característiques de les funcions polinòmiques.

Definició.
La seva expressió general es
f ( x)  an x n  an 1x n 1  ...  a1x  a0 , amb ai  IR

Les funcions polinòmiques no presenten cap tipus de problema de domini, i per


tant el seu domini sempre és IR.

La gràfica de les funcions polinòmiques.


Les gràfiques de les funcions polinòmiques són com filferros que es van
doblegant tantes vegades com el seu grau.

Atura't i pensa...
Tots els polinomis de grau senar tenen, almenys, una arrel real. Perquè?

Sabies que...?
Nessie, el monstre del llac Ness, té forma de cúbica (funció polinòmica de grau
3)
4.1.1
Determina quines gràfiques no corresponen a funcions polinòmiques:

4.1.2
De quin grau pot ser el polinomi que té associada la següent gràfica:

A) 2 B) 3 C) 4 D) 5

4.1.3
Observa la següent pauta:

El nombre màxim de regions formades connectant n punts en una circumferència


és
f ( n) 
1 4
24

n  6n3  23n 2  18n  24 

a) Quin és el grau d'aquest polinomi?


b) Quin és el seu coeficient principal?
c) Determina el nombre màxim de regions formades connectant 5 punts en una
circumferència. Comprova el resultat dibuixant els punts i comptant a mà les
regions.
d) Quants punts hauríem de dibuixar per a tenir 99 regions?
El criteri del coeficient principal.

Donat el polinomi
f ( x)  an x n  an 1x n 1  ...  a1x  a0

Si el grau del polinomi és senar:

Si el grau del polinomi es parell:


4.2 La gràfica de les funcions polinòmiques.

Exercici resolt.
Representa gràficament la funció f ( x)  x3  5x 2  3x  2 , mitjançant una taula
de valors X-Y.

Solució.
Completem una taula de valors X-Y

x  2  f (2)  (2)3  5(2)2  3(2)  2  32


x  1  f (1)  (1)3  5(1)2  3(1)  2  7
x  0  f (0)  (0)3  5(0)2  3(0)  2  2
...

Marquem aquests punts en un sistema de coordenades X-Y:

No passarem sempre tots els punts. Hem d'excloure aquells que siguin massa
grans. En aquest cas no representem (-2, -32) i (5, 17)

Ara només cal unir aquests punts, recordant que la gràfica d'una funció de tercer
grau és com Nessie, el monstre del llac Ness:
4.3 Punts notables d'una funció polinòmica.

Punts de tall amb l'eix X


Els punts de tall amb l'eix X són les solucions de l'equació f ( x)  0 .També
s'anomenen arrels del polinomi.
La clau per trobar les arrels d'un polinomi és la factorització, i si no es pot,
Ruffini. I si no es pot amb Ruffini, per computadora.

Exemple resolt.
Determina els punts de tall amb l'eix X de la funció f ( x)  x3  4 x .

Solució.
Hem de resoldre l'equació x3  4 x  0 i ho fem factoritzant el polinomi:


0  x3  4 x  x x 2  4  Traiem factor comú x
0  xx  2( x  2) Apliquem la identitat notable
a 2  b2  (a  b)(a  b)
x  0

0  xx  2( x  2)   x  2  0  x  2 Apliquem el principi del producte
 x  2  0  x  2

nul.

Els punts de tall amb l'eix X són: x  0 , x  2 i x  2


Exercici resolt.
Determina les arrels del polinomi f ( x)  x3  3x 2  x  3

Solució:
Hem de trobar les solucions de l'equació x3  3x2  x  3  0 .
x 3  3x 2  x  3  0  Hem de resoldre aquesta equació.
x ( x  3)  1( x  3)  0 
2
Agrupem factors repetits.
( x  3)( x  1)  0 
2
Tornem a agrupar factors repetits
 x  3  0  x  3
( x  3)( x 2  1)  0   2 Apliquem el Criteri del producte nul
 x  1  0  x  1
Les solucions són x  3 , x  1 , x  1

Exercici resolt.
Determina les arrels del polinomi f ( x)   x 4  4 x3  4 x 2

Solució:
 x 4  4 x3  4 x 2  0 Hem de resoldre aquesta equació.

 x2 x2  4x  4  0 Traiem factor comú  x 2
 x  x  2  0 Factoritzem x 2  4 x  4 (de cap, mentalment!)
2 2

 x 2  0  x  0

 x 2 x  2  0  ( x  2) 2  0 
2
Apliquem el criteri del producte nul.
x  2  0  x  2

Les solucions són x  0 i x  2 :
Factorització amb el mètode de Ruffini.

4.3.1
Factoritza els següents polinomis (fent servir Mètode de Ruffini):
a) 2 x 3  12 x 2  2 x  60
b) 3x 3  12 x 2  3x  12
c)  2 x 4  18x 3  54 x 2  62 x  24

4.3.2
Determina els punts de tall i representa gràficament les següents funcions:
a) f ( x)  3x 3  9 x 2  3x  9
b) f ( x)  2 x 3  2 x 2  18x  18
c) f ( x)  x 4  3x 3  21x 2  43x  60
4.4 Funcions polinòmiques presentades en forma factoritzada.
Exemple resolt.
Representa gràficament la següent funció polinòmica:

P( x)  x  2x  1x  3

Solució:

Determinem els seus punts de tall amb l’eix X, aplicant el mètode del producte
A  0
nul: A  B  0  
B  0
En el nostre cas:
 x  2

P( x)  x  2x  1x  3  0   x  1
x  3

Ara prenem punts de prova entre aquests valors per saber si estan per sobre o per
sota de l’eix X

Amb aquesta informació representem la funció:

Si volem, a més a més, obtenir la forma algèbrica d’aquesta funció en forma de


suma de potències d’x, haurem de fer el producte dels seus factors:

P( x)  x  2x  1x  3  x3  2 x 2  5x  6
4.4.1
Representa les següents funcions polinòmiques.

a) f ( x)  xx  3x  1

b) f ( x)   x 2 2 x  3x  1

c) f ( x)  x  2 x  2
2
4.5 Determinació de f. polinòm. donats els punts de tall amb l'eix X.

4.5.1
Determina les funcions polinòmiques donat el seu grau i els seus punts de tall
amb els eixos. Escriu les funcions en forma de suma de potències d’x.

a) Grau del polinomi: 3, Punt de tall amb l’eix Y: 8, Punts de tall amb l’eix X: -
2,1 i 2

30

20

10

-6 -4 -2 2 4 6

-10

-20

b) Grau del polinomi: 3, Punt de tall amb l’eix Y: -9, Punts de tall amb l’eix X: -
3,1 i 3
40

20

-6 -4 -2 2 4 6

-20

-40

c) Grau del polinomi: 4, Punt de tall amb l’eix Y: 24,


Punts de tall amb l’eix X: -3 , -2, 1 i 2
80

60

40

20

-4 -3 -2 -1 1 2 3

d) Grau del polinomi: 5, Punt de tall amb l’eix Y: 45,


Punts de tall amb l’eix X: -5, -3, -1, 1 i 3

400

200

-6 -4 -2 2 4

-200

-400
4.5.2
Aquí tens quatre gràfiques:
A B

C D

... i quatre expressions algèbriques:


i) f ( x)  x3  3x 2  9 x ii) f ( x)  x3  x 2  8x  12
iii) f ( x)  2 x  3x 2  x3 iv) f ( x)  9  9 x  x 2  x3

Associa cada gràfica amb la seva expressió algèbrica.


Problema resolt.
Si el punt (5,15) pertany a la gràfica de la funció polinòmica
f ( x)  ax( x  4)( x  1)
a) Determina el valor de a .
b) Determina les arrels del polinomi, és a dir, els seus punts de tall amb l'eix X.
c) Representa gràficament la funció.

Solució:

a) (5,15) pertany a la gràfica de la funció és equivalent a dir que


1
15  f (5)  a  5(5  1)(5  1)  a  5  4  6  15  120a  a 
8
b) Hem de resoldre l'equació f ( x)  0
1 / 8  0 mai
x  0
1 
0  f ( x)  x( x  4)( x  1)  
8 x  4  0  x  4
 x  1  0  x  1

Les arrels són x  0 , x  4 , x  1 .

c) La funció és de grau 3, una cúbica, amb coeficient principal positiu, que talla
l'eix X en  1 , 0 , 4 .
1
A més a més, f (0)  0(0  4)(0  1)  0
8
Amb això ja tenim una visió aproximada de com és la seva gràfica:

4.5.3
Si el punt (2,3) pertany a la gràfica de la funció polinòmica
f ( x)  a( x  3)( x  2)2
a) Determina el valor de a .
b) Determina les arrels del polinomi, és a dir, els seus punts de tall amb l'eix X.
c) Representa gràficament la funció.
4.6 Transformacions de funcions polinòmiques.

Transformacions de funcions cúbiques.

4.7.1
Sobre la base de la funció g ( x)  x3 , i els seus punts notables, representa
gràficament les següents funcions, sense fer cap taula de valors, fent servir el
mètode de les translacions:

a) f ( x)  ( x  2)3  1 b) f ( x)  ( x  1)3  1 c) f ( x)  ( x  3)3  2


4.7.2
Sobre la base de la funció g ( x)  x3 , i els seus punts notables, representa
gràficament les següents funcions, sense fer cap taula de valors, fent servir el
mètode de les translacions:

a) f ( x)  x  2  3 b) f ( x)  x  2  4 c) f ( x)  x  3  2
3 3 3
4.7 Repàs de funcions polinòmiques.

Primera llista de repàs de funcions polinòmiques.

4.7.1
Determina els punts de tall de la funció f ( x)  3x  4 amb els eixos.

4.7.2
Determina la funció de primer grau f ( x)  ax  b que passa pels punts
P  (2 , 11) i Q  (1 , 2)

4.7.3
Donada la funció de segon grau f ( x)  x 2  5x  4 , determina:
a) Punt de tall de la funció amb l’eix Y.
b) Punts de tall de la funció amb l’eix X.
c) Vèrtex de la paràbola.
d) Eix de simetria.
e) Representa gràficament la funció sense fer taula de valors, assenyalant els
valors anteriors:
4.7.4
Determina la funció de segon grau f ( x)  ax 2  bx  c que passa pels punts
P  (1,5) , Q  (2 , 2) , R  (0,2)

4.7.5
Determina els punts de tall A i B entre la funció f ( x)  3x  2 i la funció
g ( x)  2 x 2  3x  5

4.7.6
a) Factoritza la funció f ( x)  x 4  x 3  12 x 2  28x  16
b) Determina el punt de tall de la funció amb l’eix Y.
c) Determina els punts de tall de la funció amb l’eix X.

4.7.7
Determina l’única funció polinòmica f (x) de tercer grau que té associada la
següent gràfica, si sabem que f (1)  18 . Escriu la funció com a suma de
potències d’x.
4.7.8
a) Escriu una funció polinòmica de grau igual o superior que no tingui cap punt
de tall amb l’eix X.
b) Escriu una funció polinòmica de grau igual o superior a 3 que només tall l’eix
X en el punt x = 4 .

4.7.9
Donada la funció f ( x)  x 3  ax 2  bx  1 , determina el valor dels paràmetres a
i b si sabem que f (2)  3 i f (1)  9

Segona llista de repàs de funcions polinòmiques.

4.7.10
Determina els punts de tall de la funció f ( x)  2 x  1 amb els eixos.

4.7.11
Determina la funció de primer grau f ( x)  ax  b que passa pels punts
P  (3,5) i Q  (2,10)

4.7.12
Donada la funció de segon grau f ( x)  2 x 2  4 x  1 , determina:
a) Punt de tall de la funció amb l’eix Y.
b) Punts de tall de la funció amb l’eix X.
c) Vèrtex de la paràbola.
d) Eix de simetria.
e) Representa gràficament la funció sense fer taula de valors, assenyalant els
valors anteriors:
4.7.13
Determina la funció de segon grau f ( x)  ax 2  bx  c que passa pels punts
A  (2,1) , B  (1, 2) C  (0,1)

4.7.14
Determina els punts de tall A i B entre la funció f ( x)  3x  2 i la funció
g ( x)  2 x 2  x  3

4.7.15
Factoritza la funció f ( x)  x 4  6 x 3  9 x 2  4 x  12

4.7.16
Donada la funció polinòmica f ( x)  2 x 3  14 x  12
a) Determina el punt de tall de la funció amb l’eix Y.
b) Determina els punts de tall de la funció amb l’eix X.

4.7.17
Determina l’única funció polinòmica f (x) de grau 3 que talla l’eix X pels punts
(2,0) (-3,0) i (0,0) i a més a més compleix f (1)  8 . Escriu aquesta funció com a
suma de potències d’x.

4.7.18
Donada la funció f ( x)  3x 2  ax  1 , determina el valor del paràmetre a si
sabem que f (3)  36

4.7.19
5x  6
Resol l’equació x 2 
x2
Tercera llista de repàs de funcions polinòmiques.

4.7.20
Factoritza f ( x)  x3  9 x 2  23x  15

4.7.21
Resol l’equació x 4  6 x3  4 x 2  54 x  45  0

4.7.22
Determina una funció polinòmica de grau 3 que talla l’eix X als punts 3, 5 i -2.
Representa aquesta funció com a suma de potències d’x.

4.7.23
Determina la funció polinòmica de grau 3 que talla l’eix X pels mateixos punts
3, 5 i -2 però que a més a més passa pel punt ( 2 , -36).

4.7.24
Determina una funció polinòmica de grau 4 que només tingui com a punts de tall
amb l’eix X els valors x = 2 i x = -3.

4.7.25
Determina una funció polinòmica de grau 4 que té la següent gràfica associada:

4.7.26
Determina el valor de k per a que la funció f ( x)  2 x 3  4 x 2  k x  1 compleixi
f (2)  5

4.7.27
Determina els valors de k per a que la funció
f ( x)  x 4  k x 3  2 x 2  (k 2  1) x  2
passi pel punt ( 1 , -2 )

4.7.28
Determina els valors a i b per a que la funció f ( x)  x 3  a x 2  bx  7
compleixi f (1)  9 i f (2)  11
Quarta llista de repàs de funcions polinòmiques.

4.7.29
Amb l’ajuda de la calculadora científica, avalua la següent funció al valor x
donat. Escriu el resultat amb 2 decimals.
f ( x)  3x 2  2 x  3
x  0.86

4.7.30
Determina el vèrtex V=(x,y) de la gràfica de la funció f ( x)  3x 2  2 x  1

4.7.31
Determina els punts A i B d’intersecció entre la paràbola f(x) i la recta g(x).
f ( x)  x 2  2 x  4
g ( x)  3x  1

4.7.32
Determina els punts de tall amb els eixos associats a la funció polinòmica de
tercer grau.
f ( x)  3x 3  12 x 2  12 x  48

4.7.33
Determina la funció f(x) = ax2 + bx + c que passa pels punts P, Q i R.

P = ( -3 , -16 ) Q=(0,2) R=(1,4)


4.7.34
Determina els punts de tall entre les funcions p(x) i q(x)

p( x)  x 3  6 x 2  x  42
q ( x)   x 2  x  2

4.7.35
Determina la funció polinòmica de tercer grau f(x) donats els seus tres punts de
tall amb l’eix X i un valor de la seva taula. Escriu aquesta funció com a suma de
potències d’x.

Punts de tall:{ 2 , -1 , -4 }, Punt de tall amb l’eix Y: 24


5 Les funcions racionals.

Ara és un bon moment per repassar...


Fraccions algebraiques i equacions racionals. (Tema 12 del Llibre d'Àlgebra)

Definició de funció racional.


La seva expressió general d’una funció racional és
p( x)
f ( x)  on p(x) i q(x) són polinomis.
q( x)

5.1 Les funcions de proporcionalitat inversa.

Les funcions de proporcionalitat inversa apareixen també quan estudiem...


Les hipèrboles (Tema 6 del Llibre de Geometria Analítica)

La funció de proporcionalitat inversa és la funció


k
f ( x)  , amb k  0
x

Gràfica de la funció de proporcionalitat inversa.


La seva gràfica és una hipèrbola equilàtera, els eixos de la qual són els eixos de
coordenades.

Les característiques d’una funció racional són aquestes:


- El seu domini és tota la recta real menys el valor x = 0: Dom f  x  0
- Té simetria respecte a l’origen de coordenades, o sigui, és una funció
imparella.
- La funció és decreixent si k > 0, i les seves branques se situen en el primer i
tercer quadrant.
- La funció és creixent si k < 0, i les seves branques se situen en el segon i quart
quadrant.
- No té punts de tall amb els eixos, encara que la gràfica s’hi aproxima tant com
vulguem en prolongar-la indefinidament:
La funció "aterra" en l'eix X quan x es fa més i més gran.
La funció "s'enlaira" en l'eix Y quan x s'apropa a 0.
Però la gràfica mai arriba a tocar cap eix.
Exemple resolt.
Suposem que y varia de forma inversament proporcional a x, i que y  3 si
x  8 . Determina la funció representa-la gràficament.

Solució:
k
y  f ( x)  Escrivim el model de la funció de proporcionalitat inversa.
x
k
3 Substituïm y per 3 i x per 8
8
3 8  k Aïllem la k
24
y Escrivim la k obtinguda a la funció.
x
Per representar-la, construïm i omplim una taula de valors X-Y:

5.1.1
Escriu l’equació de la funció que té per gràfica una hipèrbola com la de la figura
amb el centre de simetria desplaçat al punt (2,-1).

5.1.2
Una aixeta amb un cabal de 8 litres/min triga 42 minuts en omplir un dipòsit.
Quant trigaria si el cabal fos de 24 litres/min? Escriu la funció cabaltemps.
Exercici resolt.
La base b d'un paral·lelogram d'àrea fixa varia de forma inversament
proporcional a la seva l'altura h, i b = 12 cm si h = 8 cm. Determina b quan h = 3
cm.

Solució:
k
b Escrivim la funció de proporcionalitat inversa amb lletres.
h
k
12  Substituïm pels valors donats a l'enunciat.
8
k  96 Trobem la constant k resolent l'equació resultant.
96
b Ja tenim la fórmula de relació.
h
96
b Substituïm el valor h donat
3
b  32 Calculem el resultat.

La base mesura 32 cm quan l'altura mesura 3 cm.


5.2 Domini de definició d’una funció racional.

Domini d'una funció racional.


Les funcions racionals presenten problemes de domini quan s'anul·la el
denominador:

Dom f  IR  q( x)  0

Exemples:
x2  6x  5
a) f ( x)   Dom f  IR    1
x 1

x2  9
b) f ( x)   Dom f  IR  3 
x3

x3
c) f ( x)   Dom f  IR   1,  4 
x  5x  4
2
5.3 Punts de tall amb l'eix X.
Determinar els punts de tall amb l'eix X equival a resoldre una equació racional
igualada a zero. El denominador passa multiplicant i se'n va, però pot donar
problemes (vegeu el segon exercici resolt).

Exercici resolt.
x2  2x  3
Determina els punts de tall amb l'eix X de la funció f ( x)  .
x5

Solució:
x2  2x  3
Hem de resoldre l'equació f ( x)  0
x5
x 2  2 x  3  0( x  5)  0 El denominador "passa" multiplicant a zero, bye
bye!...
0  x 2  2 x  3  ( x  3)( x  1) Queda el numerador, que hem de
factoritzar, (o aplicar la fórmula)
x  3  0  x  3
0  ( x  3)( x  1)   Apliquem el Principi del Producte Nul.
 x  1  0  x  1

Les solucions són x  3 i x  1 . Les comprovem:


32  2  3  3 0
x  3  f (3)   0
35 2
(1) 2  2  (1)  3 0
x  1  f (1)   0
(1)  5 2
Exercici resolt.
x 1
Determina els punts de tall amb l'eix X de la funció f ( x)  .
x2  1

Solució:
x 1
Hem de resoldre l'equació f ( x)  0
x2  1
x  1  0( x 2  1)  0 El denominador "passa" multiplicant a zero, bye bye!...
x  1 Queda el numerador, que només té solució x  1

Però al comprovar aquesta solució veiem que queda fora del domini de definició
de la funció:
11 0
x  1  f (1)  2  no està definit (és una discontinuïtat evitable)
1 1 0

Aquesta funció no té cap punt de tall amb l'eix X:


5.4 Simplificació de funcions racionals.

Exemple resolt.
2x  x2
Simplifica la funció f ( x) 
x2  x  2

Solució:
Factoritzem numerador i denominador:
2x  x2  x x  2  x
f ( x)  2   si x  2
x  x  2 ( x  2)( x  1) x  1

La funció no està definida per a x  2 i per a x  1

Exemple resolt.
x2 x4
Simplifica la funció f ( x)  
3x  12 x 2  4

Solució:
Factoritzem:
x2 x4 x2 x4
f ( x)   2  
3x  12 x  4 3x  4 x  2x  2

I "tatxem" els factors repetits:


1 1 1
f ( x)    si x  2 i x  4
3 x  2 3( x  2)

La funció no està definida per a x  2 i per a x  4

Si tatxes factors, no oblidis mai d'afegir la coletilla: "La funció no està definida
per a..."
5.5 Asímptotes verticals.
Direm que la recta vertical x  a és una asímptota vertical de la gràfica de la
p( x)
funció racional f ( x)  quan f (x)   quan x  a
q( x)

És a dir, quan la funció f (x) es fa més i més gran sense límit quan x a mida que
x s'apropa més i més a x  a .

P( x)
En general, si tenim una funció racional f ( x)  , i es compleix que
Q( x)
P(a)  k  0 i Q(a)  0 , llavors lim f ( x)   .
x a

Per determinar a quin cas correspon, avaluarem la funció en dos punts propers a
x  a , un per la dreta i un altre per l’esquerra.

Cal diferenciar els quatre possibles casos:

A) lim f ( x)   C) lim f ( x)  
x a x a

lim f ( x)   lim f ( x)  
x a  x a 

B) lim f ( x)   D) lim f ( x)  
x a x a
lim f ( x)   lim f ( x)  
x a x a 
5.5.1
Relaciona quina gràfica correspon a cada funció:
1 1 x 1 1 x
a) f ( x)  b) f ( x)  c) f ( x)  d) f ( x) 
x 1 x 1 x x
x 1 x 1
e) f ( x)  f) f ( x) 
x 1 x 1

5.5.2 Exercici del Youtube


Estudia les asímptotes verticals de les següents funcions:
x3 3x 1
a) f ( x)  b) f ( x)  2 c) f ( x) 
x2 x 1 9  x2
Solució: https://youtu.be/121nx9DXMqo (Susi Profe)

5.5.3
Determina els següents límits:
x3  4x 2  x x2  9 x 4  3x3  x 2  x  4
a) lim b) lim c) lim
x 0 x 2  4x x 3 x 2  3 x x  1 x 1

5.5.4 Exercici del Youtube (en anglès)


Determina les asímptotes verticals de les següents funcions:

x 1 x 2  3x  10 4
a) f ( x)  b) f ( x)  c) f ( x) 
x  5x  6
2
x2  4 x 1
2

Solució: https://youtu.be/V137qmDN9Qw (NancyPi)


Exemple.
x2
Les rectes x  3 i x  3 són asímptotes verticals de f ( x) 
x2  9

Mètode pràctic.
p( x)
Les asímptotes verticals de f ( x)  les trobarem quan s'anul·la el denominador:
q( x)

q ( x)  0

i no s'anul·la el numerador: p( x)  0 .

Si s'anul·la numerador i denominador tenim una "indeterminació 0/0" i pot ser


una asímptota o no. S'han d'estudiar amb mètodes específics.
Exercici resolt.
x
Determina i representa gràficament les asímptotes de la funció f ( x) 
x 9
2

Solució:
Factoritzem numerador i denominador:

Observem que 9 és un quadrat i recordem la identitat notable: a 2  b2  (a  b)(a  b)


x x
f ( x)  2 
x  9 ( x  3)( x  3)

El denominador s'anul·la per x  3 i x  3 , i amb aquests valors no s'anul·la el


numerador, per tant no estem davant de cap indeterminació 0/0. Les asímptotes de la
funció són les rectes verticals x  3 i x  3 :
5.6 Indeterminació 0/0. Discontinuïtats evitables.
x3
Determina i representa gràficament les asímptotes de la funció f ( x) 
x2  9

Solució:
Factoritzem numerador i denominador:
x3 x 3
f ( x)  
x  9 ( x  3)( x  3)
2

El denominador s'anul·la per x  3 i amb aquest valor no s'anul·la el numerador. Per


tant, aquesta funció té una asímptota vertical en x  3 .

Però, Atenció! Per a x  3 també s'anul·la el numerador, i per tant estem davant d'una
indeterminació 0/0:
33 0 0
f (3)   
(3  3)(3  3) 0  6 0

Simplifiquem numerador i denominador "tatxant" el factor repetit x  3 .

La funció resultant és

Aquesta funció no està definida per a x  3 , però no hi ha asímptota. Hem resolt la


indeterminació.

Si el factor s'elimina, la asímptota se'n va, però a la gràfica li queda una petita
marca, un petit foradet, com si fos una espinilla mal curada en la cara. I no ens
hem d'oblidar de dibuixar-lo.
Discontinuïtats evitables.
P( x)
Donada una funció racional f ( x)  , si tenim que P(a)  0 i Q(a)  0 ens trobem
Q( x)
0
en un cas d’indeterminació del tipus .
0

Per a resoldre aquesta indeterminació, factoritzem numerador i denominador,


simplifiquem i calculem el límit de la expressió simplificada:

P( x) P( x) /( x  a)
f ( x)   si x  a
Q( x) Q( x) /( x  a)

El resultat pot ser una discontinuïtat evitable:


lim f ( x)  lim f ( x)  k
x  x 

Exercici resolt.
2 x 2  5x  3
Determina les discontinuïtats evitables de la funció f ( x) 
x 3

Solució:
2 x 2  5 x  3 (2 x  1)( x  3) 2 x  1 si x  3
f ( x)   
x 3 ( x  3) no def si x  3

Avaluem el valor de la funció resultat al punt x  3  2  3  1  7


Aquesta funció presenta una discontinuïtat evitable al punt 3,7  , que es representa amb
un petit cercle buit, com si la gràfica tingués una espinilla:ç

Exercici resolt.
x 4  16
Determina les discontinuïtats evitables de la funció f ( x) 
x2

Solució:
x 4  16 ( x 2  4)( x  2)( x  2)
f ( x)    ( x 2  4)( x  2) si x  2
x2 ( x  2)

x  2  y  (22  4)(2  2)  32
Aquesta funció presenta una discontinuïtat evitable al punt 2,32

Les discontinuïtats evitables estan fora del domini de la funció.


5.7 Comportament d’una funció racional en l’infinit.

Esquema principal.
p( x)
Donada una funció racional f ( x)  , amb p(x) i q(x) polinomis,
q( x)

a) Si grad (q)  grad ( p) , llavors lim f ( x)  0


x 

2 x 3 5 x 2  4 x  4
Exemple: f ( x)  , lim f ( x)  0
3x 5  3x 2  7 x 

b) Si grad (q)  grad ( p) , llavors lim f ( x)  k ,


x 
on k és el quocient entre el coeficient del monomi de grau màxim del numerador
i el coeficient del monomi de grau màxim del denominador.

4x 2  2x  1 4
Exemple: f ( x)  , lim f ( x) 
3x  3
2 x  3

Tant en el cas 1 com en el 2, parlarem d’asímptotes horitzontals.

c) Si grad ( p)  grad (q) , llavors lim f ( x)  


x 

En particular, si grad ( p)  grad (q)  1 , tindrem una asímptota obliqua:


quan x creix, els valors de la funció tendeixen als valors d’una recta y  ax  b ,

5.7.1 Exercici del Youtube (en anglès)

Estudia el comportament en l’infinit de les següents funcions:

x 1 x2  1 x2  1
a) f ( x)  2 b) f ( x)  2 c) f ( x) 
x 1 3x  1 x3
Solució: https://youtu.be/qJrrZQgSkO8 (NancyPi)
Determinació d’una asímptota obliqua mitjançant límits:
, i b  lim  f ( x)  ax
f ( x)
a  lim
x  x x 

Exercici resolt 1.
2x 2  1
Determina l'asímptota obliqua de la funció: f ( x) 
x2
Solució:
f ( x) 2x 2  1
a  lim  lim 2 2
x  x x  x  2 x

 2x 2  1   4x  1 
b  lim  f ( x)  a x  lim   2 x   lim  4
x  x 
 x2  x   x  2 
Per tant, la funció f(x) té com a asímptota obliqua la recta y  2 x  4

Exercici resolt 2.
x2 1
Determinar l’asímptota obliqua de la funció f ( x) 
2x  3

Solució:
f ( x) x2 1 x2 1 1
a  lim  lim  lim 2   0.5
x  x x  x( 2 x  3) x  2 x  3 x 2
 3 
 x2 1 1  *   x 1  3
b  lim  f ( x)  ax   lim   x   lim  2  2   3  0.75
x  x  2 x  3 2  x   2 x  3 
  
2 4
 
x2 x  3 x 2  1  x2 x  3
1 1
x2  1 1 x2  1 2 2
*  x   
2x  3 2 2x  3 2x  3 2x  3
2 3 3
x2  1  x2  x  x  1
 2 2  2
2x  3 2x  3

1 3
Per tant, l’asímptota obliqua és y  x   0.5 x  0.75
2 4
Determinació d’una asímptota obliqua mitjançant una divisió de polinomis.

Una forma alternativa de trobar-la és fent la divisió del numerador entre el


denominador.

Exercici resolt.
x 2  2x  2
Determineu l’asímptota obliqua de la funció f ( x) 
x3
Solució:

Per tant , l’asímptota obliqua de la funció és y  x  1

Exercici resolt.
3x 2  7 x  5
Determineu l’asímptota obliqua de la funció f ( x) 
x 1

Solució:

La funció té l’asímptota obliqua y = 3x + 4

5.7.2
Comportament de les següents funcions en l’infinit:
1 4 x 2  3x  1 2x2
a) f ( x)  2 b) f ( x)  c) f ( x ) 
x 1 2x2  7 x3
3
x
d) f ( x) 
x  12
5.8 Estudi d’una funció racional (sense derivació).

Taula resum de l’estudi d’una funció racional (sense derivació).

a) Domini de definició.
Una funció racional està definida sempre que no s’anul·li el denominador.
Dom( f )    x | Q( x)  0
b) Discontinuïtats evitables.
Factoritzem numerador i denominador i eliminem els factors repetits.
Aquest factors repetits són les discontinuïtats evitables de la funció.

c) Punt de tall amb l’eix Y.


Avaluem la funció per a x  0  f (0)

d) Punts de tall amb l’eix X.


Una funció racional té un zero (punt de tall amb l’eix X) en x  a si
P(a)  0 i Q(a)  0 , és a dir, s’anul·la el numerador però no el denominador en
a.
La funció es perfectament contínua en aquest punt.

e) Asímptotes verticals.
Una funció racional té una asímptota vertical en x  a si P(a)  0 i Q(a)  0 ,
és a dir, s’anul·la el denominador però no el numerador.
També poden aparèixer asímptotes verticals a algunes de les indeterminacions
0/0.
En particular, la funció no està definida en x = a, i per tant té una discontinuïtat
en aquest punt.

f) Comportament d’una funció racional en l’infinit.


Podem trobar infinits, asímptotes horitzontals i asímptotes obliqües. Vegeu
apartat 2.3.

g) Gràfica.
Nota importat:La gràfica de la funció mai pot creuar una asímptota vertical.
Però pot creuar tantes vegades com vulgui una asímptota obliqua o una
asímptota horitzontal.
Quadre resum de les asímptotes de les funcions racionals.

num( x)
f ( x) 
den( x)

Asímptota vertical

den( x)  0 i num( x)  0 (després de factoritzar)

Asímptota horitzontal

grau num( x)  grau den( x)

Asímptota obliqua

grau num( x)  grau den( x)  1


5.8.1 Exercicis.
Estudi complet de les següents funcions, amb tots els elements indicats a la taula
anterior:

a) Punts de tall amb l’eix X.


b) Punt de tall amb l’eix Y.
c) Asímptotes verticals.
d) Comportament de la funció en l’infinit.
e) Gràfica (esquemàtica) de la funció amb la informació dels apartats anteriors.

x3 x 2  6x  8
a) f ( x)  b) f ( x) 
x  4x  5
2
x 2  4x  5
( x  3) 2 x 3  2 x 2  5x  6
c) f ( x)  d) f ( x) 
x 1 x2  x  6
x  7 x 2  20 x  96
3
x 2  11x  28
e) f ( x)  3 f) f ( x)  3
x  9 x 2  14 x  24 x  6 x 2  x  30
x3  3x 2  25 x  21
g) f ( x) 
x 2  3x  10
5.9 Problemes PAU amb funcions racionals (sense derivació).

5.9.1
3x 3  5 x  2
Trobeu les asímptotes de la funció f ( x) 
x 2  4x  5
PAU CAT TEC SET 2010 2.1

5.9.2
x 2  4x  1
Trobeu el domini i les asímptotes de la funció definida per f ( x) 
x 1
PAU CAT TEC JUNY 2006 1.4

5.9.3
ax  b
Sabem que la funció f ( x)  passa pel punt (2, –5) i que les rectes x = 1 i
cx  1
y = 2 en són les asímptotes vertical i horitzontal, respectivament. Calculeu a, b i c.

PAU CAT CCSS SET 2018 3.6 Sol.: SC pàg. 422

5.9.4
Donada la funció següent:
x2
f ( x) 
x 2 1
a) Determineu-ne el domini i els valors de x per als quals el signe de la funció f és
negatiu.
b) Determineu les asímptotes horitzontals i verticals de la funció f.

PAU CAT CCSS SET 2010 2.6

5.9.5
Trobeu les equacions de les asímptotes verticals i obliqües de la funció
x 3  5x 2
g ( x)  2
2 x  18
PAU CAT CCSS SET 1998 2.4
6 Les funcions radicals.

Ara és un bon moment per repassar...


Expressions i equacions amb radicals (Tema 13 del Llibre d'Àlgebra)

6.1 Característiques de les funcions radicals.

Definició.
Anomenem funció radical quan la variable independent x es troba davall el signe
radical.
f ( x)  n g ( x)

Recorda:
La funció f ( x)  x 2 no és bijectiva, la seva inversa f 1 ( x)  x només està definida
en 0, .

La funció f ( x)  x3 és bijectiva, la seva inversa f 1 ( x)  3 x està definida en tot IR.

Memoritza les gràfiques de les funcions x 2 , x , x3 , 3


x
6.2 La gràfica de les funcions radicals.

Exercici resolt.
Representa gràficament la funció arrel quadrada y  f ( x)  x

Construeix una taula de valors X-Y. Observa que la arrel quadrada no està definida per
a nombres negatius.

Exercici resolt.
Representa gràficament la funció arrel quadrada y  f ( x)  4 3 x  4

Construeix una taula de valors X-Y. Observa que la arrel cúbica de nombres negatius sí
està definida.

6.2.1
Representa gràficament la funció f ( x)  x  1
6.2.2
Representa gràficament la funció arrel quadrada: y  f ( x)  x mitjançant la
calculadora científica:

6.2.3
Representa gràficament la funció arrel cúbica y  f ( x)  3 x mitjançant la calculadora
científica:
6.3 Domini de definició de les funcions radicals.

Si l’índex n és parell, el domini és 0,


si l’índex n és imparell, el domini és tota la recta real.

Exercici resolt.
Calcula el domini de la funció f ( x)  x  1 i representa-la.

Solució:
x  1  0  x  1  Dom f   1,

Representem la seva gràfica amb l'ajut d'una petita taula de valors:

x f ( x)  x  1
-1 0
0 1
1 2  1.41
2 3  1.73

6.3.1
Determina el domini de definició de les següents funcions:

a) f ( x)  2 x  3 b) f ( x)  x 2  16
1
c) f ( x)  ( x  3)( x  5) d) f ( x) 
x
x2 x4
e) f ( x)  f) f ( x) 
x3 x  5x  6
2

g) f ( x)  x 2  x  6 h) f ( x)  ln( x)
i) f ( x)  sin( x)

6.3.2 (Repàs)
Resol les següent equacions:
a) x  1  x  7 b) 3 x  1  11  2 x c) x  2  2x  1  4x

6.3.3 M
Determina el domini de la funció
x 1 x2
f ( x)  
 x3 x4
6.4 Punts notables de la gràfica de funcions amb arrels quadrades.

6.4.1
Per a cadascuna de les funcions següents,
a) Determina:
- L’extrem P de la gràfica.
- El punt de tall Ix amb l’eix X.
- El punt de tall Iy amb l’eix Y.

b) Passa aquests punts a un sistema de coordenades X-Y i amb la seva ajuda representa
gràficament la funció.

a) f ( x)  x  2  3 b) f ( x)  x  3  5 c) f ( x)  x  5  4
d) f ( x)   x  3 e) f ( x)  4  x g) f ( x)  2  3 x  2
h) f ( x)  2 2  2 x  4 i) f ( x)  2  5  3x j) f ( x)  2 2 x  4
k) f ( x)  5  3x  3
6.5 Arrels quadrades com a parametritzacions de la circumferència.

Les gràfiques de les funcions de la forma y  f ( x)   a 2  x 2 son trossos de


circumferència. En efecte:


y   a2  x2  y 2   a2  x2  a
2
2
 x2  x2  y 2  a2  x2  y 2  a

I, aplicant Pitàgores, la gràfica forma part de la circumferència de centre ( 0 , 0 ) i radi


a.

Per exemple:
La gràfica de y  f ( x)  4  x 2 és la part superior de la circumferència de centre
( 0 , 0 ) i radi 2:

La gràfica de y  f ( x)   4  x 2 és la part inferior de la circumferència de centre


( 0 , 0 ) i radi 2:

I amb totes dues tenim parametritzada tota la circumferència:


6.5.1
a) Dedueix dues funcions f1 ( x) i f 2 ( x) per a parametritzar la circumferència de centre
l’origen (0,0) i radi 2:

x -2 -1.5 -1 -0.5 0 0.5 1 1.5 2


f 1 ( x)
f 2 ( x)

b) Dedueix dues funcions f1 ( x) i f 2 ( x) per a parametritzar la circumferència de centre


l’origen (0,0) i radi 3:

x -3 -2.5 -2 -1.5 -1 -0.5 0 0.5 1 1.5 2 2.5 3


f 1 ( x)
f 2 ( x)

c) Representa gràficament els punts anteriors. Quines figures geomètriques hi


aparèixen?

6.5.2
Determina la gràfica de les següents funcions, mitjançant una taula de valors, en un
mateix sistema de coordenades:
a) y  f ( x)  9  x 2 b) y  f ( x)   9  x 2

Quina figura geomètrica hi apareix?


Exercici resolt.
Determina gràficament i algèbricament els punts d’intersecció de les següents funcions,
i comprova que els valors obtinguts son coherents:
f ( x)  25  x 2 , g ( x)  2 x  1

a) Des d’un punt de vista geomètric:


y  f ( x)  25  x 2 representa la part superior d’una circumferència de centre ( 0 , 0 ) i
radi 5.
y  g ( x)  2 x  1 representa una recta que passa pels punts (0,1) i (2,5):
Representem gràficament aquestes dues figures:

I veiem que tallen en un únic punt, aproximadament, ( 1.8 , 4.7 )

b) Anem a determinar aquest punt de forma algèbrica:


f ( x)  g ( x)  25  x 2  2 x  1 

 25  x   2x  1 
2
2
2

25  x 2  2 x  1  4 x 2  4 x  1 
2

0  4 x 2  4 x  1  25  x 2 
 4  4 2  4  5  (24)  4  496 1.827
0  5 x 2  4 x  24  x   
25 10  2.627

Comprovem les solucions obtingudes (és obligatori!):


f (1.827)  25  1.827 2  4.654

  f (1.827)  g (1.827) , la solució és acceptable
g (1.827)  2 1.827  1  4.654 

f (2.627)  25   2.627   4.254


2

  f (2.627)  g (2.627) , la solució no és


g (2.627)  2 2.627   1  4.254 

acceptable

Per a la solució acceptable: x  1.827  y  2 1.827  1  4.654 , i el punt de tall és


(1.827 , 4.654 )
6.5.3
Determina gràficament i algèbricament els punts d’intersecció de les següents perelles
de funcions, i comprova que els valors obtinguts son coherents:


 y  f ( x)  9  x 2 
 y  f ( x)  16  x 2 
 y  f ( x)  9  x 2
a)  b)  c) 

 y  g ( x)  x  1 
 y  g ( x)  x  5 
 y  g ( x)   x  4

6.5.4
Determina, aproximadament, les solucions de l’equació

49  x 2  25  x 2  3

de forma gràfica, sense resoldre les equacions.

6.5.5
Determina els punts d’intersecció entre la circunferència de centre (0,0) i radi 4 i la recta
que passa pels punts ( -5 , 1) i ( 6 , 3 ), de forma gràfica i algèbrica. Comprova la
coherència dels resultats obtinguts.

6.5.6
Determina els valors aproximats de les solucions de l’equació

25  x 2  16  x2  5

(sense resoldre l’equació), només interpretant gràficament les seves parts.


6.5.7
Determina algèbricament (resolent una equació) els dos punts de tall A i B de l’esquema
següent.

6.5.8
Determina, de geomètrica i algèbrica, el punt d’intersecció entre la semicircumferència
superior de centre (0,0) i radi 10 i la recta que passa pels punts (0,-4) i (2,2). Comprova
que els dos resultats obtinguts són coherents.
6.6 Transformacions de la funció arrel quadrada.

6.6.1
Escriu cada funció com a composició de la funció base f ( x)  x , i representa-les
gràficament. Indica el seu domini i el seu recorregut.

a) g ( x)  2 x b) g ( x)  (1 / 2) x c) g ( x)   x
d) g ( x)  2 x e) g ( x)   x f) g ( x)   2 x
g) g ( x)    x h) g ( x)  2  x i) g ( x)  x  3
j) g ( x)  x  3 k) g ( x)   x  4  2 l) g ( x)   x  2
m) g ( x)    x  1  2
6.6.2 Exercici del Youtube

Translacions.

a) Representar gràficament la funció g ( x)  x  2 a partir de la gràfica de la funció


f ( x)  x
Solució: https://youtu.be/94V8PxtPrR0 (A partir del minut 12:1)

b) Representar gràficament la funció g ( x)  x 2  2 a partir de la gràfica de la funció


f ( x)  x 2
Solució: https://youtu.be/94V8PxtPrR0 (A partir del minut 14:51)

c) Representar gràficament la funció g ( x)  ( x  2) 2  3 a partir de la gràfica de la


funció f ( x)  x 2
Solució: https://youtu.be/94V8PxtPrR0 (A partir del minut 44:06)

6.6.3 Exercici del Youtube

Simetries.

a) Mirar el vídeo https://youtu.be/i69D7XfaX5M (fins al minut 13:35)

per repassar els conceptes estudiats a classe.

b) Representar gràficament la funció f ( x)   x  1  2

Solució: https://youtu.be/i69D7XfaX5M (A partir del minut 13:35)

6.6.4 Intersecció de gràfiques (Primer grau vs. Arrel quadrada)

Determina els punts de tall entre les gràfiques de les dues funcions de cada apartat.
i) Visualment, representant les dues gràfiques.
ii) Algèbricament, resolent l’equació.
Comprova que les solucions obtingudes són coherents.

a) f ( x)  2 x  3 , g ( x)  x  3 b) f ( x)  x  2 , g ( x)  3 x
c) f ( x)  7  2 x , g ( x)  x  2 d) f ( x)  3x  4 , g ( x)   x  2
e) f ( x)  2 x  1 , g ( x)  2  x
6.7 Transformacions de funcions amb arrels cúbiques.

6.7.1
Transformació d'arrels cúbiques. Representa les següents funcions:

a) f1 ( x)  3 x  3 b) f 2 ( x)  3 x  2 c) f 3 ( x)  3 x  5
6.7.2
Representa les següents funcions:

a) f1 ( x)  3 x  4 b) f 2 ( x)  3 x  2 c) f 3 ( x)  3 x  5
6.7.3
Representa les següents funcions:

a) f1 ( x)  3 x  4  2 b) f 2 ( x)  3 x  4  3
c) f 3 ( x)  3 x  2  1 d) f 3 ( x)  3 x  3  2
6.7.4
Representa les següents funcions:

a) f1 ( x)  3 x b) f 2 ( x)  3  x
c) f 3 ( x)  3  x  3 d) f 3 ( x)  3  x  4  3
6.7.5
Associa cada funció amb la seva gràfica:
a) f ( x)  x  2  2 b) f ( x)  x  2  2
c) f ( x)   x  2  2 d) f ( x)   x  2  2

6.7.6
Donada la gràfica de la funció arrel quadrada:

Determina les expressions algèbriques associades a les següents funcions:

a) b) c)
El coneixement del comportament de les funcions pot ser la clau per resoldre problemes
d'equacions semblen molt complicats, com en el següent exemple:

Problema.
Donats els nombres reals a, b, c  0 , resol l'equació:

a  bx  b  cx  c  ax  b  ax  c  bx  a  cx

Romania 1974 Solució: PA #4.1


6.8 Repàs d’equacions i funcions amb arrels quadrades (I).

6.8.1
Resol la següent equació: x2 5

6.8.2
Resol la següent equació: 2  3 x  2  11

6.8.3
Resol la següent equació: x 8  x  2

6.8.4
Resol la següent equació: x  4  2  2x  7  4

6.8.5
Representa les següents funcions:

a) f1 ( x)  x  2
b) f 2 ( x)   x
c) f 3 ( x)   x  3
6.8.6
Donada la funció f ( x)  2 x  4  3
a) Determina l’extrem P de la gràfica de la funció.
b) Determina el punt d’intersecció amb l’eix X.
c) Determina el punt d’intersecció amb l’eix Y.
d) Representa els punts anteriors i amb la seva ajuda representa gràficament la
funció.

6.8.7
a) Representa gràficament les següents funcions, i assenyala el seu punt de tall:

f 1 ( x)  x  2  1 f 2 ( x)  2 x  3

b) Determina algèbricament el punt de tall de les dues gràfiques anteriors.


6.8.8
a) Representa gràficament les següents funcions, i assenyala els seus punts de tall:

f 1 ( x)  3  x  1 f 2 ( x)   x  2  4

b) Determina algèbricament els punts de tall de les dues gràfiques, i comprova que els
resultats obtinguts són coherents amb els trobats a l’apartat anterior.
6.9 Repàs d’equacions i funcions amb arrels quadrades (II).

6.9.1
Resol les següents equacions:

a) x2 3 b) 2 x  1  8  14

c) x4  x2 d) x  4 1  x  5  2

6.9.2
Representa les següents funcions:

a) f1 ( x)  x  3 b) f 2 ( x)    x c) f 3 ( x)   x  4

6.9.3
Donada la funció f ( x)  3x  8  5

a) Determina l’extrem P de la gràfica de la funció.


b) Determina el punt d’intersecció amb l’eix X.
c) Determina el punt d’intersecció amb l’eix Y.
d) Representa els punts anteriors i amb la seva ajuda representa gràficament la funció.
6.9.4
a) Representa gràficament les següents funcions, i assenyala el seu punt de tall:

f 1 ( x)  x  4  3 f 2 ( x)  2 x  1

b) Determina algèbricament el punt de tall de les dues gràfiques anteriors.

6.9.5
Representa les següents funcions:

a) f1 ( x)  3 x b) f 2 ( x)  3 x  4  3 c) f 3 ( x)  3  x
6.9.6
Representa gràficament les següents funcions, i determina els seus punts de tall, de
forma gràfica i algèbrica:

f 1 ( x)  3 x  1  2 f 2 ( x)  x  1

6.9.7
Representa gràficament una circumferència de radi 5 i centre en l’origen, mitjançant una
taula de valors de almenys 8 punts.
6.10 Repàs d’equacions i funcions amb arrels quadrades (III).

6.10.1
Resol les següents equacions:
a) x  5  2 b) 7  2 x  2  15 c) x9  x7
d) x  4  3  2 x  1  4

6.10.2
Representa les següents funcions:
a) f1 ( x)  x  1 b) f 2 ( x)   x c) f 3 ( x)   x  2

6.10.3
Donada la funció f ( x)  3x  6  2
a) Determina l’extrem P de la gràfica de la funció.
b) Determina el punt d’intersecció amb l’eix X.
c) Determina el punt d’intersecció amb l’eix Y.
d) Representa els punts anteriors i amb la seva ajuda representa gràficament la funció.
6.10.4
Representa les següents funcions:
a) Representa gràficament les següents funcions, i assenyala el seu punt de tall:

f 1 ( x)  x  1  2 f 2 ( x)  2 x  1

b) Determina algèbricament el punt de tall de les dues gràfiques anteriors.

6.10.5
a) Representa gràficament les següents funcions, i assenyala el seu punt de tall:

f 1 ( x)  2  x  2 f 2 ( x)   x  1  4

b) Determina algèbricament els possibles punts de tall de les dues gràfiques.


7 Les funcions exponencials.
7.1 Característiques de les funcions exponencials.

Ara és un bon moment per repassar...


Exponencials i logaritmes (Tema 6 del Llibre d'Àlgebra)
Les equacions exponencials i logarítmiques (Tema 14 del Llibre d'Àlgebra)

Definició de la funció exponencial.


Definim la funció exponencial de base a (sempre suposant a > 0 ) com
f ( x)  a x
Gràfica de la funció exponencial.

Si a>1 és creixent Si a<1 és decreixent.

La gràfica sempre és positiva i mai arriba a tocar l'eix de les x, tot i que s'hi aproxima
tant com es vulgui (això significa que l'eix de les x és una asímptota horitzontal de la
gràfica).

Propietats de les exponencials.


a0  1
a1  a
ax y  ax  a y
x
 1  1 x 1 1
 x      a , en particular, a 
a  a a
a b  ab
x x x

Per a tot n>1, n


a  a1 / n

En particular, fent a  e  2.71828183 , tenim la funció exponencial de base e, o


simplement « exponencial » f ( x)  e x

Per a calcular exponencials amb la calculadora científica (els models antics) farem
. Per exemple, per a calcular  7 
6
servir la tecla
7.1.1 Repàs.
Associa cada gràfica amb la seva expressió algèbrica.

1
i) f ( x)  ii) f ( x)  3x iii) f ( x)  x 2  4
x

iv) f ( x)  x 3 v) f ( x)  x3  x 2  6 x

A B C

D E

7.1.2 Exercici del Youtube


https://youtu.be/GNvr3h1AB_g (Xavi Mates)

7.1.3 Exercici del Youtube


https://youtu.be/A2N0CW_AL-Y (Unicoos)
7.2 La gràfica de les funcions exponencials.

7.2.1
Completa la taula i representa gràficament.
a) f ( x)  4 x

b) f ( x)  0.5x
c) f ( x)  1.25x

d) f ( x)  2 x / 2
7.2.2
Representa gràficament les següents funcions.
a) f ( x)  4  2 x b) f ( x)  5  2 x

x x
1 1
c) f ( x)  4    d) f ( x)  2   
2 2
x 1
x2 1
e) f ( x)  3  2 2 f) f ( x)  4    2
2
7.2.3
Representa gràficament les següents funcions:

x
1
a) f1 ( x)  2 x b) f 2 ( x)  3 x c) f 3 ( x)     0.5 x d) f 4 ( x)  0.1x
2
7.3 Determinació de funcions exponencials.

Exercici resolt.
Determina la funció exponencial f ( x)  a x que s'adapta a la següent gràfica:

f ( x)  a x , i sabem que passa pel punt (2,25) , per tant:


25  f (2)  a 2  a  25  5
Per tant, la funció buscada és f ( x)  5x

Exercici resolt.
Determina la funció exponencial f ( x)  a x que s'adapta a la següent gràfica:

f ( x)  a x , i sabem que passa pel punt (3,1/ 8) , per tant:


3
1 1 1 1
 f (3)  a3  3     a3  a 
8 2 2 2
Per tant, la funció buscada és f ( x)  (1/ 2) x

7.3.1
Determina la funció exponencial f ( x)  a x que s'adapta a cada gràfica:
7.3.2 Exercici resolt.
La següent gràfica mostra una funció del tipus f ( x)  k a x

a) Determina els valors de a i k .


b) Avalua la funció per a x  3 .

Solució:
a) El punt (0,5) pertany a la gràfica de la funció, per tant f (0)  5
5  f (0)  k a0  k  1  k  k  5

El punt (2,45) pertany a la gràfica de la funció, per tant f (2)  45


45  f (2)  5 a 2  45  5 a 2

Obtenim una equació de segon grau que hem de resoldre:


45
45  5 a 2   a 2  9  a 2  a  3
5

Les funcions exponencials sempre tenen la base positiva, per tant ens quedem amb la
solució positiva de l'equació: a  3

I arribem a f ( x)  5  3x

Comprovem que, efectivament, aquesta funció satisfà les condicions de l'enunciat:


f (0)  5  30  5  1  5
f (2)  5  32  5  9  45

b) Avaluem la funció:
f (3)  5  33  5  27  135
7.3.3
La següent gràfica mostra una funció del tipus f ( x)  k a x

Determina els valors de a i k .

7.3.4
The diagram shows a sketch of the graph of y  ab x

The curve passes through the points A (0.5, 1) and B (2, 8)


The point C (-0.5, k) lies on the curve.
Find the value of k
7.3.5 Exercici resolt.
La següent gràfica mostra una funció del tipus exponencial f ( x)  k a x . Determina
aquesta funció.

Solució:
Ens interessa treballar amb el valor de la funció x  0 , per tant, farem una
transformació d'aquesta funció: Traslladem la seva gràfica una unitat cap a l'esquerra.

Els punts ara són: (0 , 5) i (3 , 320) . Trobem la funció:


g ( x)  k a x
5  g (0)  k a 0  k  k  5
320
320  g (3)  5 a3   a3  64  a3  64  a3  a  4
5

Per tant, la funció és g ( x)  5  4 x

I ara desfem la translació que hem fet, és a dir, ara traslladem la gràfica una unitat cap a
la dreta. Recordem que "traslladar una unitat cap a la dreta" és realitzar la composició
f ( x)  g ( x  1)

I, ara sí, la nostra funció és


f ( x)  g ( x  1)  5  4 x 1

Si volem obtenir la seva expressió de la forma f ( x)  k a x fem una mica d'àlgebra:


5
f ( x)  5  4 x 1  5  4 x  41   4 x
4
7.4 Modelitzant amb funcions exponencials.

7.4.1
Creixement d’una població cel·lular (Aplicació de la funció exponencial).

Considerem que un bacteri es divideix cada 40 segons en dos nous bacteris, cadascun
dels quals es torna a dividir en dos més cada 40 segons i així successivament, és a dir,
que la població de bacteris es dobla cada 40 segons, sempre que les condicions del
cultiu bacteriològic no es modifiquin.

a) Completa la següent taula

Segons Bacteris
0 1
40
80
120
160

b) Representa mitjançant una fórmula matemàtica el nombre de bacteris que hi ha en


cada moment en funció del temps t en segons.
c) Quants bacteris hi haurà en el cultiu al cap de 10 minuts?
d) Quant de temps haurà de passar per a obtenir un cultiu de 1000000 de bacteris ?
e) Quant de temps haurà de passar per a obtenir un cultiu de 109 bacteris ?

7.4.2
La grandària d’un cert cultiu de bacteris es multiplica per 2 cada 30 minuts. Si suposem
que el cultiu té inicialment 5 milions de bacteris, quantes hores trigarà a tenir 320
milions de bacteris?

7.4.3
La grandària d’un cert cultiu de bacteris es multiplica per 2 cada 20 minuts. Si al cap de
3 hores el cultiu té 576 milions de bacteris, quants n’hi havia a l’instant inicial?
7.5 Transformacions de funcions exponencials.

Exercici resolt.
Utilitza la gràfica de la funció f ( x)  3x per representar la gràfica de la funció
g ( x)  3x 1

Solució:
Estudiem la funció g (x) com una composició de funcions: g ( x)  f ( x  1)
Aquesta composició de funcions es correspon, gràficament, a una translació horitzontal
d'1 unitat cap a l'esquerra.

Exercici resolt.
Utilitza la gràfica de la funció f ( x)  2 x per representar la gràfica de la funció
g ( x)  2 x  3

Solució:
Estudiem la funció g (x) com una composició de funcions: g ( x)  f ( x)  3
Aquesta composició de funcions es correspon, gràficament, a una translació vertical de
3 unitats cap avall.
7.5.1
Per a cada funció, determina:
a) Punt de tall amb l’eix Y.
b) Possible punt de tall amb l’eix X.
c) Valors f (1) o f (1)
d) Asímptota horitzontal.
Amb els punts anteriors representa gràficament la funció.

Funció Model Transformació geomètrica

a) f ( x)  2 x  3

b) f ( x)  0.59 x  5
f ( x)  a x  b

c) f ( x)  3 x  1

d) f ( x)  2  3 x

e) f ( x)  0.5  2 x
f ( x)  k  a x

f) f ( x)  3  0.2 x

g) f ( x)  2 x

h) f ( x)  0.8 x
f ( x)  a x

i) f ( x)  3  2 x  4

j) f ( x)  0.4 x  2 (Composició de dos o més)

k) f ( x)  4  3 x  5
7.5.2
Representa gràficament les següents funcions, indicant:
Asímptotes verticals i horitzontals.
Punts de tall amb els eixos.
Punts notables.

a) f ( x)  4 x b) f ( x)   log x c) f ( x)  log 3 x d) f ( x)   log 3 x


e) f ( x)  ln x  1 f) f ( x)  2  log 3 x g) f ( x)  2  log 3 x h) f ( x)  log 2 x  4
i) f ( x)  log 4 x  1 j) f ( x)  ln x  3 k) f ( x)   ln x  2 l) f ( x)  5  ln x
x
m) f ( x)  3  ln x n) f ( x)  1  e
7.6 Punts de tall amb funcions exponencials.

Exercici resolt.
Determinar el punt d’intersecció de les gràfiques de les funcions

f ( x)  2  5 x  3 g ( x)  3  5 x  1

Solució:

Igualem les dues fórmules:

f ( x)  g ( x)  2  5 z  3  3  5  x  1  2  5 z  3  3  5  x  1  0 
1
2  5z  3  x  4  0
5
Fem el canvi de variable p  5 x
1
2  p  3  4  0
p
Multiplicant per p obtenim una equació de segon grau:
2  p2  3  4 p  0  2  p2  4 p  3  0 
4  2 2  4  2  (3) 4  4  24 4  28 p  -0.581139
p   
22 4 4 p  2.58114

Descartem la primera solució perquè una exponencial no pot ser negativa.


log( 2.58114)
p  5 x  2.58114  x  log 5 (2.58114)   0.589169
log( 5)
y  f (0.589169)  2  50.589169  3  2.16228

El punt d’intersecció és ( 0.589 , 2.162 )


Exercici resolt.
Determina, aproximadament, els punts de tall entre les gràfiques de les funcions

f ( x )  e  x i g ( x)  x  2

Solució:
Representem gràficament les dues funcions:

Observem que les dues gràfiques tenen un punt de tall: x  0.5 aproximadament, i que
aquest serà el seu únic punt de tall possible (una és creixent i l'altra decreixent).

Si volem més precisió, podem tirar de calculadora:

f ( x)  g ( x)  e  x  x  2  e  x  x  2  0

x e x  x  2
-0.5 0.149
-0.4 -0.108
-0.45 0.0183
-0.425 -0.045

Prenent sempre l'interval en el què canvia el signe anem ajustant més i més el valor de
la solució. Amb una computadora podem obtenir tants decimals com volguéssim:

x = -0.44285440100238858314132799999933681971626212937347968471773...

7.6.1
Determinar, aproximadament, els punts de tall entre les gràfiques de les funcions
f ( x )  2 x i g ( x)  4  x
7.7 Repàs de funcions exponencials.

7.7.1
Determina la funció exponencial f ( x)  a x si sabem que passa pel punt P  ( 1 , 3 )

7.7.2
Determina la funció exponencial f ( x)  a x si sabem que passa pel punt P  ( 7 , 3 )

7.7.3
Determina la funció exponencial f ( x)  a x si sabem que passa pel punt
P  ( 2.5 , 0.1 ) . Podem esperar un valor a major o menor que 1?

7.7.4
Donada la funció f ( x)  (0.7) x , determina les coordenades dels punts P, Q, R i S que
pertanyen a la seva gràfica:

on P = ( -2 , ? ) , Q = ( -1 , ? ) , R = ( 1.55 , ? ) i S = ( 3.9 , ? )
7.7.5
Donada la funció f ( x)  (3.2) x , determina les coordenades dels punts P, Q, R i S que
pertanyen a la seva gràfica:

on P = ( ? , 2 ) , Q = ( ? , 0.25 ) , R = ( ? , 0.8 ) i S = ( ? , 0.1 )

7.7.6
Resol les següents equacions:

a) 4  3 x  5  0 b) 3  4 x  6  0 c) 5 x2  3  5 x1  8  0

d) e 2 x  e x  6  0 e) 2 x  2 2x  0

7.7.7
La població d’un país (mesurada en milions d’habitants) creix exponencialment de la
forma f (t )  30  e 0.01t , on la variable t representa els anys transcorreguts des de l’any
base 1980.

a) Completa la següent taula:

Any 1980 1981 1982 1985 1990 1995


Població

b) En quin any la població duplicarà la de 1980?

c) En quin any la població duplicarà la de 1990?


7.7.8
A partir de la gràfica de la funció f ( x)  2 x , representa gràficament les següents
funcions:

f 1 ( x)  2 x  1 , f 2 ( x)  2 x  3 , f 3 ( x)  2 x 1 , f 4 ( x)  2 x 2 ,
f 5 ( x)  2  x , f 6 ( x)  2  x  1 , f 7 ( x)  log 2 x

7.7.9
En el següent gràfic estan representades les funcions f ( x)  1.2  0.93 x i
g ( x)  0.75  1.4 x
Associa cada funció amb la seva gràfica corresponent, i determina el punt A=(x,y) de
tall de les dues gràfiques.
7.7.10
Determina el punt A d’intersecció de les gràfiques de les funcions f ( x)  18.6 x i
g ( x)  2 x 1 . Assenyala prèviament la gràfica que correspon a cada funció.

7.7.11
Determina el punt A d’intersecció de les següents funcions: f ( x)  1200  (0.9) x i
g ( x)  750  (1.13) x . Assenyala prèviament la gràfica que correspon a cada funció.

Problema D
Determina totes les solucions de l'equació:

10x  11x  12x  13x  14x

Solució: PA #6.5
8 Les funcions logarítmiques.

Ara és un bon moment per repassar...


Exponencials i logaritmes (Tema 5 del Llibre d'Àlgebra)
Les equacions exponencials i logarítmiques (Tema 14 del Llibre d'Àlgebra)

8.1 Característiques de les funcions logarítmiques.


Definició.
Definim el logaritme de base a, i escriurem f ( x)  log a ( x) com la funció inversa de
l'exponencial de base a: g ( x)  a x .
y  f ( x)  log a ( x)  x  a y

Propietats dels logaritmes.


log a ( xy )  log a ( x)  log a ( y)
log a ( x / y)  log a ( x)  log a ( y)
log a ( p)
log a ( x n )  n log a ( x) , log a (n p ) 
n

Logaritmes especials.
Existeixen dos casos especials de logaritme:
- El logaritme en base 10 s'anomena logaritme decimal i es representa com a log( x) ,
sense cap tipus de base. El trobem a la calculadora científica a la tecla
- El logaritme en base e = 2.7182818... s'anomena logaritme natural o logaritme
neperià, i es pot representar com a ln(x) . El trobem a la calculadora científica a la tecla

Gràfica de les funcions logarítmiques.


Les funcions logarítmiques són inverses de les funcions exponencials. Recorda (Apartat
1.6) com són les gràfiques de les funcions inverses: Simètriques respecte la bisectriu
yx
Domini de definició de les funcions logarítmiques.
Les funcions exponencials sempre donen valors positius, per tant els logaritmes només
estan definits per als valors positius.

Exercici resolt.

Determina el domini de la funció f ( x)  ln 4  x 2 
Solució:

Dom f  4  x 2  0   4  x2  0  2  x  2  Dom f    2 , 2 

Ara és un bon moment per repassar...


Les inequacions de segon grau. (Apartat 15.2 del Llibre d'Àlgebra)
8.2 La gràfica de les funcions logarítmiques.

8.2.1
Representa gràficament les següents funcions:

a) f1 ( x)  ln( x) b) f 2 ( x)  log 2 ( x) c) f3 ( x)  log 0.2 ( x)


d) f 4 ( x)  log( x) e) f5 ( x)  log 0.6 ( x)
Exercici resolt.
Determinar el punt d’intersecció de les gràfiques de les funcions
f ( x)  log 5 ( x  1)  2 g ( x)   log 5 ( x  2)  1

Solució:
Igualem les dues fórmules:
f ( x)  g ( x) 
log 5 ( x  1)  2   log 5 ( x  2)  1 
log 5 ( x  1)  log 5 ( x  2)  3 
log 5 ( x  1)( x  2)  3 
( x  1)( x  2)  5 3 
x 2  x  2  125 
x 2  x  127  0 
1  1  4  1  (127) 1  1  4  1  (127) 1  509 - 10.7805
x   
2 2 2 

La primera solució no és acceptable perquè genera un logaritme d’un nombre negatiu.


La solució correcta és x = 11.7805

y  f (11.7805)  log 5 (11.7805  1)  2  -0.416888

El punt d’intersecció és ( 11.781 , -0.417 )


8.3 Transformacions de funcions logarítmiques.

Ara és un bon moment per repassar l'Apartat 1.4

Exercici resolt.
Representa gràficament la funció g ( x)  log 5 x  2

Solució:
En aquest cas veiem que la transformació realitzada és la següent: g ( x)  f ( x)  2 , que
es correspon gràficament a una translació vertical de 2 unitats cap amunt:

Exercici resolt.
Representa gràficament la funció g ( x)  log10 x  3

Solució:
En aquest cas veiem que la transformació realitzada és la següent: g ( x)  f ( x  3) , que
es correspon gràficament a una translació horitzontal de 3 unitats cap a la dreta:
Exercici resolt.
Representa gràficament la funció
g ( x)  log 4 x  2  3

Solució:
Primer pas: Representem la funció base f ( x)  log 4 ( x) .

Sabem que el logaritme de 1 és sempre 0, i que el logaritme en base 4 de 4 és 1, per tant,


aquesta funció passa pels punts (1,0) i (4,1).

Sabem també que tot logaritme té asímptota vertical x  0 .

Amb aquesta informació ja en tenim prou per representar-la:

Segon pas: Interpretem la funció g ( x)  log 4 x  2  3 com g ( x)  f ( x  2)  3 , i per


tant la seva gràfica estarà situada 2 unitats a la dreta i 3 unitats cap amunt. Movem el
punts de referència i l'asímptota vertical.
Tercer pas: Unim els nous punts de referència, a mà alçada i sense por, seguint amb la
vista la funció base:

8.3.1
Representa gràficament les següents funcions, de forma aproximada i seguint les
indicacions de l'exercici resolt anterior. Determina el seu domini i el seu recorregut.

a) f ( x)  log 6 x  1  5 b) f ( x)  log 5 x  1  3
c) f ( x)  log 6 x  3  5 d) f ( x)  log 2 x  1  3

e) f ( x)  log 4 x  1  4 f) f ( x)  log 5 x  1  1

g) f ( x)  log 4 x  2  1 h) f ( x)  log 6 x  2  1
8.3.2
Associa cada expressió algèbrica amb la seva representació gràfica.

a) b)

c) d)

e) f)

i) f ( x)  log 3 x  2 ii) f ( x)   log 3 x iii) f ( x)   log 3 x  2

iv) f ( x)  log 3 ( x  1) v) f ( x)  log 3 (1  x) vi) f ( x)   log 3 ( x)


9 Les funcions trigonomètriques.

Trobareu el desenvolupament d'aquest tema a...


Les funcions trigonomètriques (Tema 3 del Llibre de Trigonometria)
10 Les funcions definides a trossos.
10.1 Concepte de funció definida a trossos.
Definició de funció definida a trossos.
Les funcions definides a trossos són aquelles en les quals el valor de la funció s’obté de
diferent manera segons l’interval de la variable independent que s’està considerant.
Una funció definida a trossos té diferents fórmules o expressions algebraiques per a
distints intervals de la variable independent.
Per obtenir la gràfica d’una funció definida a trossos han de representar-se tots els seus
trams d’acord amb la funció present en cadascun; també cal prestar una atenció especial
al valor que adopta la funció en els extrems de cada interval.

Exercici resolt.
Determina l'expressió algèbrica associada a la següent gràfica:

A l'esquerra de x  4 la funció és constant igual a 4:


y  4 si x  4

Entre x  4 i x  2 la gràfica es recta, i per tant la funció és de primer grau:


y  ax  b
Amb pendent -1 y  (1) x  b
i val -2 quan x  0  2  (1)  0  b  b  2
La funció és y   x  2

A la dreta de x  2 , la gràfica es recta, i per tant la funció és de primer grau: y  ax  b


Amb pendent 1 y  1x  b
i val -4 quan x  2  4  1  2  b  b  6
La funció és y  x  6
Ara recollim tot en una única expressió:
 4 si x  4

f ( x)   x  2 si  4  x  2
 x  6 si x  2

10.1.1
Relaciona cada funció amb la seva gràfica.

 x  4 si x  1 2 x  3 si x  0  x  4 si x  0
1. f ( x)   2. f ( x)   3. f ( x)  
3x si x  1  x  4 si x  0 2 x  4 si x  0

3x  1 si x  1 3x  2 si x  1  3x  1 si x  1
4. f ( x)   5. f ( x)   6. f ( x)  
 5 si x  1  x  2 si x  1  5 si x  1

10.1.2
Determina l'expressió algèbrica associada a cada gràfica:

a) b)
10.1.3
Representa gràficament la següent funció:
 3 si x  3
f ( x)  
2 x  5 si x  3

Determina f (4)  f (0)  f (3) 


10.1.4
Representa gràficament la següent funció:
 x  5 si x  2
f ( x)  
 2 x  1 si x  2

Determina f (3)  f (4)  f (2) 


10.1.5
Representa gràficament les següents funcions.

 4 si x  2
 2 x  1 si x  2 
a) f ( x)   b) f ( x)   x  2 si - 2  x  2
 x  4 si x  2  2 x  4 si x  2

 2 x si x  4
  6 si x  2
c) f ( x)   x si - 4  x  0 d) f ( x)  
( x  1) si x  -2
4

4  x si x  0
2

 1 2 x si x  3
 si x  4 
e) f ( x)   x  4 f) f ( x)   3
 4 si x  4  si x  3
x

x
 si x  4  x  3 si x  1
2 

g) f ( x)    x si - 4  x  2 h) f ( x)  x  1 si x  1
4

 x  2 si x  2 
  4 x si x  1

10.3 La funció part entera.
Definim la part entera d'un nombre x com el nombre enter més gran que sigui igual o
menor que x. Es denota per
f ( x)  x

Per exemple: 2.1  2 , 4.57  4 , 8  8 ,  2  2 ,  3.4  4

La notació x  va ser introduïda per Gauss al 1808. Nosaltres farem servir la notació
x  , que va ser introduïda per Kenneth E. Iverson als anys 60 del segle passat.

Observa que equival a tatxar la part decimal només si el nombre es positiu. Si el


nombre és negatiu no és veritat:
Per exemple:  7.15  7 ,  7.15  8

Gràfica de la funció part entera.

La funció "part entera" apareix sovint a les competicions matemàtiques:


Vegeu Tema 8 del llibre "Álgebra Problem Solving"

La funció "part entera" s'utilitza molt en aritmètica:


Vegeu Tema 13 del llibre "Teoría de números"
Problema. M
Determina (si és que existeixen) polinomis f (x) , g (x) y h(x) que compleixin:

 1 si x  1

f ( x)  g ( x)  h( x)  3x  2 si  1  x  0
 2 x  2 si x  0

Putnam 1999 Solució: PA #7.3


11 La funció valor absolut.

11.1 El concepte de valor absolut.

Definició de valor absolut.


El valor absolut d’un nombre real, |x|, coincideix amb el nombre si aquest és positiu o
nul, o bé amb el seu oposat, −x, si és negatiu; així doncs, pot considerar-se com una
funció definida a trossos, ja que aquesta operació exigeix canviar el signe del seu
argument:

 x si x  0
f ( x)  x  
 x si x  0

Gràfica del valor absolut.


La seva gràfica és:

La gràfica del valor absolut té una propietat molt important: És una funció que no es del
tot suau: Punxa en x=0, (passa la mà i notaràs com se't clava).

Propietats del valor absolut:.


a) a  0
b) a   a
c) a  b  a  b (Desigualtat triangular)
d) a  b  a  b
a a
e) 
b b
11.2 Transformacions de la funció valor absolut.

11.2.1
Donada la gràfica de la funció valor absolut:

Determina les expressions algèbriques associades a les següents funcions:

a) b) c)

11.2.2
Representa gràficament les següents funcions:

a) f ( x)  x  2 b) f ( x)  x  2
c) f ( x)  3x  2 d) f ( x)  x 2  1
e) f ( x)  x 2  4 x  3 f) f ( x)  ( x  3)( x  2)( x  7)
x
g) f ( x)  x  x h) f ( x)  (“funció signe”)
x
1
i) f ( x)  sin( x) j) f ( x)  sin( x) 
2
k) f ( x)  ln( x) l) f ( x)  x
Problema resolt. Una funció peculiar.
La funció f ( x)  x  3  4 x  1  x  8  6 x  1

té un comportament molt sorprenent, propi d’una funció definida a trossos: és constant


igual a 1 entre x=5 i x=10:

Normalment pensem que una funció definida “algèbricament” (amb polinomis i arrels)
que sigui constant al llarg de tot un interval només pot ser la pròpia funció constant.
Aquesta funció podria ser un bon antiexemple. Ara bé, estudiant més detingudament
aquesta funció podem observar que apareixen valors absoluts.

El primer terme x  3  4 x  1 en realitat és l’arrel quadrada del quadrat de


2  x  1 , que és una forma “algèbrica” d’expressar un valor absolut, i el valor absolut
sí que és, en realitat, una funció definida a trossos:


x  3  4 x 1  2  x 1 
2
 2  x 1  
2  x  1 si x  5

 x  1  2 si x  5
2  x 1  0  2  x 1  4  x 1  5  x

De la mateixa manera, el segon terme es pot escriure com l’arrel quadrada del quadrat
de 3  x  1 , que és una forma “algèbrica” d’expressar un valor absolut:


x  8  6 x 1  3  x 1 
2 3  x  1 si x  10
 3  x 1  

 x  1  3 si x  10
Per tant,
f ( x)  x  3  4 x  1  x  8  6 x  1  2  x  1  3  x  1

Ara es veu més fàcilment que entre 5 i 10 aquesta funció “cancel·la” el terme x 1 , i
per tant és constant:

5  x  10  f ( x)  x  1  2  3  x  1  1
En realitat, qualsevol funció de la forma f ( x)  a  g ( x)  b  g ( x) tindrà aquest
peculiar comportament.
Atenció amb el tatxar quadrats i arrels!

x2  x  x  x
2

x 2  x amb domini  ,  x   x amb domini 0,


2

11.2.3
Donades les funcions f ( x)  4  x 2 i g ( x)  x ,
a) Determina f  g i el seu domini de definició.
b) Determina g  f i el seu domini de definició.
c) Representa gràficament les dues funcions anteriors.
12 Problem-solving amb funcions.
12.1 El llenguatge de les funcions.
12.1.1
Si f és una funció tal que, per a tot valor enter de x es compleix que
f ( x  1)  2 f ( x)  2002

i, a més, sabem que f (2005)  2008 , llavors f (2004) és:

(A) 2004 (B) 2005 (C) 2008 (D) 2010 (E) 2016

CANGUR 2005 N4 #8

12.1.2
1 1
Quina de les funcions següents satisfà la condició f    per a tot x  0 ?
 x  f ( x)

(A) f x   (B) f x   (C) f x   1 (D) f x   (E) f x   x 


2 1 1 1 1
x x 1 x x x
CANGUR 2012 N4 #8

12.1.3
La gráfica de la función f, definida para todos los números reales, está formada por dos
semirrectas y un segmento, como se muestra en la figura.

¿Cuál es el conjunto de soluciones de la ecuación f  f  f x   0

(A) {-4, 0} (B) {-8, -4, 0} (C) {-12, -8, -4, 0} (D)  (E) {-16, -12, -8, -4, 0}

CANGURO 2003 N5 #20

12.1.4
En Víctor ha dibuixat la gràfica d’una funció f : IR → IR, formada per dues semirectes i
un segment, com es veu en la figura.

Quantes solucions té l’equació f  f  f x   0 ?

(A) 4 (B) 3 (C) 2 (D) 1 (E) 0


CANGUR 2013 N4 #15
12.1.5
Juan ha dibujado la gráfica de una función f : R → R, formada por dos semirrectas y un
segmento, como se ve en la figura.

¿Cuántas soluciones tiene la ecuación f  f  f x   0 ?

(A) 4 (B) 3 (C) 2 (D) 1 (E) 0

CANGURO 2013 N6 #17

12.1.6
Quin dels quatre quadrants no conté cap punt de la gràfica de la funció lineal
f ( x)  3.5x  7 ?

(A) I (B) II (C) III (D) IV (E) Tots els quadrants en contenen.

CANGUR 2017 2n Batx #10

12.1.7
En un sistema de coordenadas cartesianas rectangulares, ¿qué cuadrante NO contiene
puntos de la gráfica de la función f ( x)  3.5x  7 ?
(A) Primer cuadrante (B) Segundo cuadrante (C) Tercer cuadrante
(D) Cuarto cuadrante (E) Todos los cuadrantes contienen puntos de esa gráfica

CANGURO 2017 N6 #8

12.1.8
Una función f tiene la siguiente propiedad f (2 x  1)  3  4 x 2  6 x  2 f (1) . ¿Cuánto
vale f (2) ?
(A) 10 (B) 9 (C) 8 (D) 7 (E) 6

CANGURO 2017 N6 #14


12.1.9
Un jarrón se llena completamente de agua, a un ritmo constante. La gráfica muestra la
altura h del agua en función del tiempo t.

¿Cuál de las siguientes podría ser la forma del jarrón?

CANGURO 2018 N6 #12

12.1.10
Un gerro s’omple d’aigua fins a dalt a un ritme constant. La gràfica mostra el nivell h de
l’aigua en funció del temps t.

Quina de les formes següents podria tenir aquest gerro?

CANGUR 2018 2n Batx. #12

12.1.11
Sea f una función tal que f ( x  y)  f ( x)  f ( y) , para todos los enteros x e y. Si
1
f (1)  , entonces el valor de f (0)  f (1)  f (2)  f (3) es
2
1 3 5 15
(A) (B) (C) (D) (E) 6
8 2 2 8
CANGURO 2018 N6 #24

12.1.12
 2010 
Soit f : IR*  IR une fonction vérifiant: x  0, 2 f ( x)  3 f    5 x . Combien
 x 
vaut f (6) ?
(A) 993 (B) 1 (C) 2009 (D) 1013 (E) 923
KANGOUROU 2010 S-3 #23
12.1.13
Soit f une fonction définie sur les réels strictement positifs et vérifiant
 2015 
f ( x)  2 f    4 x . Quelle est la valeur de f (5) ?
 x 
(A) 860 (B) 1005 (C) 1068 (D) 2010 (E) 2016

KANGOUROU 2010 S-4 #21

12.1.14
Soit S la fonction définie sur IR par S ( x)  7  x si x  0,7 et, pour tout x,
S ( x  7)  S ( x) . Combien vaut S(2019) ?

(A) 0 (B) 3 (C) 4 (D) 6 (E) 7

KANGOUROU 2019 J-3 #14


12.2 Problem-Solving amb la gràfica de la funció.

12.2.1
La Paula mira al mòbil l'aplicació del temps, que mostra les temperatures mínimes
previstes per als pròxims cinc dies.

Quina forma té el gràfic de l'evolució de la temperatura?

CANGUR 2021 4t ESO #4

12.2.2
La figura mostra el gràfic d’una funció f : [−5; 5] → IR. Quantes solucions diferents té
l’equació f ( f ( x))  0 ?

(A) 8 (B) 4 (C) 7 (D) 2 (E) 6

CANGUR 2021 2n Batx. #21


13 Llistes de repàs amb funcions.
13.1 Gràfiques Segon grau-Arrel quadrada-exponencial-logarítmica.
13.1.1
i) Representa la funció f (x) com a composició de funcions d’una funció base
adequada.
ii) Representa gràficament la funció f (x) com a composició de transformacions.
iii) Indica, a la gràfica anterior, els punts de tall de la funció f (x) amb els eixos.
iv) Determina aquests punts de tall de forma algèbrica, i comprova que els resultats són
coherents amb els valors obtinguts a l’apartat anterior.

a) f ( x)   x 2  3 b) f ( x)  x  4  3 c) f ( x)  9  x 2  2
d) f ( x)  3 x  4 e) f ( x)   log( x  2)  3

13.1.2
i) Representa la funció f (x) com a composició de funcions d’una funció base
adequada.
ii) Representa gràficament la funció f (x) com a composició de transformacions.
iii) Indica, a la gràfica anterior, els punts de tall de la funció f (x) amb els eixos.
iv) Determina aquests punts de tall de forma algèbrica, i comprova que els resultats són
coherents amb els valors obtinguts a l’apartat anterior.

a) f ( x)  ( x  3) 2  5 b) f ( x)   x  3  1 c) f ( x)  16  x 2  2
d) f ( x)  2 x  3 e) f ( x)  ln( x  4)  2

13.1.3
i) Representa cada funció f (x) com a composició de funcions d’una funció base
adequada.
ii) Representa gràficament la funció f (x) com a composició de transformacions.
iii) Indica, a la gràfica anterior, els punts de tall de la funció f (x) amb els eixos.
iv) Determina aquests punts de tall de forma algèbrica, i comprova que els resultats són
coherents amb els valors obtinguts a l’apartat anterior.

a) f ( x)   x 2  4 b) f ( x)  x  5  3 c) f ( x)   9  x 2  2
d) f ( x)  5 x  3 e) f ( x)   log 3  x  5
Solucions.
1.1.1 f ( x)  x 2  1
1.1.2 a) IR b) IR   4 c) IR  0,2 d) 1, e) 0,1 f) IR
1.1.3 a) (6,6) b) (3,4) c) 2 i 4 d) -2 e) f (4)  2 f) x = -3 , x = 5
1.1.4 a) 1 , -1, 3 , 4 b) Dom   3, 3 , Im   1,4 c) 1.2 i -1.4 (aprox.)
d) -1 e) -3, 2 , 4 f) mín= ( (0,1) , màx=(3,4)
1.1.5 a) 3, 2, -2, 0 b) Dom   4,4 , Im   2,3 c) -1 i 1.8 (aprox.)
d) -2 e) mín= ( (0,2) , màx=(-4,3)
1.1.6 a) 3, -1 b) Dom   4,4 , Im   2,3
c) Dom   4,3, Im  1.5 , 4 (aprox.) d) No n'hi ha
e) ( -2 , 1 ) , ( 2, 2)
1.1.7 a) 6 , 3 , 4 , 6 b) Dom  1,8 , Im  1.2 , 6.2 (aprox.)
c) ( 2 , 4 ) , ( 5 , 5) , ( 7 , 3 )
1.2.1 a) ( f  g )(4)  f (4)  g (4)  3  2  5 b) g  f (0)  g (0)  f (0)  0  1  0
c) no definit, perquè g (1) no està definit.
g g (0) 0 f f (0) 1
d)  (0)   0 e)  (0)   no definit.
f f (0) 1 g g (0) 0
1.2.2 a) ( f  g )(4)  f (4)  g (4)  9  2  11
b) no està definit perquè g (2) no està definit.
c) ( f  g )(1)  f (1)  g (1)  3  1  3
d) no definit perquè g (0)  0
1.3.1 a)  f  g ( x)  0  x 2  4 x  3  0  ( x  1)( x  3)  0  x  1, x  3
 
b) g  f ( x)  x 2  5  0   x2  2 x  3  x2  5  0  2 x  8  0  x  4

1.3.2 f g x   0  (2 x  5)2  9  0  (2 x  5)2  9  3  2 x  5  3  1  x  4


1.4.1
 x 8  x 8
 f  g ( x) f ( g ( x))  f    3  8  x
 3   3 
g  f ( x)  g ( f ( x))  g 3x  8  (3x  8)  8  x
3

Son funcions inverses una d'altra.

2.1.1 a) Sí, a  3 , b  2 b) Sí, a  6 , b  3


c) No d) Sí, a  5 / 4 , b  1 / 4
2.1.2 a) Sí, a  1 / 3 , b  2 b) Sí, a  4 , b  1 c) No
d) No e) Sí, a  1 / 5 , b  1 / 5 f) No
g) Sí, a  3 / 2 , b  1 / 2
2.1.3 a) m1 , m3 positius, m2 , m4 negatius, m1  m3  m2  m4
b) b1 ,b2 positius, b3 ,b4 negatius, b2  b1  b4  b3
2.1.4 -2, 1/2, 3, -1/4
2.1.5 a) x + y - 4 = 0 c) 3x - 2y - 6 = 0
2
2.1.6 a) f ( x)  3x  2 b) f ( x)  x4 c) f ( x)  5x  7 d) f ( x)   x  8
5
2.1.8 F: f ( x)  2 x  1 , C: f ( x)  2 , A: f ( x)  3x  2
2.2.1

2.2.2

1 4 2 11
2.4.1 a) f ( x)  x  , 4 / 3 , -4 b) f ( x)  x  , 11 / 3 , 11/ 2
3 3 3 3
1 3 3 8
c) f ( x)  x  , 3 / 5 ,  3 d) f ( x)  x  , 8 / 5 ,  8 / 3
5 5 5 5

2.5.1 a) (-2, -11) b) ( 3 , 7 ) c) (-1,-1) d) (5 / 9,7 / 9)


1 3
2.5.2 a) f ( x)   x  4 , g ( x)  x  2 ( 3 , 5/2 )
2 2
b) f ( x)  0.5x  1 , g ( x)  2 x  3 ( 8/3 , 7/3 )
c) f ( x)  0.2 x  3 , g ( x)  3x  1 ( 5/4 , 11/4 )
d) f ( x)  2 x  3 , g ( x)  0.25x  2 ( 20/9 , -13/9 )
2.5.3 a) f ( x)  x  1 , f ( x)  2 x  11 ; (4,3)
b) f ( x)  2 x  4 , f ( x)   x  4 ;  8 / 3 , 4 / 3    2.67 , 1.33 

c) f ( x)  2 x  7 , f ( x)  3x  12 ;  19 / 5 , 3 / 5    3.8 , 0.6 

d) f ( x)  x  2 , f ( x)   x  3 ;  10 / 3 , 4 / 3    3.33 , 1.33 
1
2
2.6.1
a)

b) 4.5 c) 28 m d) 46 m e) 8 anys
2.6.2 a) f (t )  20t  800 b) 2000 litres c) 3.5 hores.
2.6.3 a) f (t )  75t b) f (t )  50t  150 c) Joan 400 km, Maria 375 km.
d) Maria: 7.01 hores, Joan: 7.52 hores.
2.6.4 a) 288€ b) 70
2.6.5 a) 2 h b) 20 km c) 15 km/h
2.6.6 a) 241.20 € b) 61.25 € c) y = 60,30x d) 5 pneumàtics
2.6.7 a) 50% b) 2016 c) 2020 d) igual que un de la ciutat B
2.6.8 a) 40 € b) 41€ c) 5 d) y= 10x
2.6.9 a) 280 € b) 400
2.6.10 a) 14 b) 180 c) b
2.6.11 Si es descarrega menys de 20 cançons, l’oferta més barata és la B.
Si es descarrega 20 cançons, qualsevol de les dues ofertes A o B.
Si es descarrega més de 20 cançons, l’oferta més barata és l’A.
2.6.12 a) c b) a c) b
2.6.13 a) b b)c
2.6.14 Preu d’una broca: 2 euros , Preu d’un paquet de tacs especials: 4 euros
2.6.15 Quota fixa d’inscripció: 10 euros , Quantitat a pagar per cada membre de l’equip: 5
euros
2.6.17 a) a b) c c) c
2.6.18 a) c b) b
2.6.19 a) c b) b c) c d) b
2.6.21 La quantitat fixa que cal pagar per enviament és de 2 euros. Per cada quilogram de pes
del paquet cal pagar 0,75 euros.
3.1.4 a) a) (0, 1) b) ( 0.27 , 0 ) i ( 3.73 , 0 ) c) (2,-3) d) x = 2

b) a) (0, -3) b) ( -2.58 , 0 ) i ( 0.58 , 0 ) c) (-1,-5) d) x = -1

c) a) (0, 1) b) ( -5.19 , 0 ) i ( 0.19 , 0 ) c) ( -2.5, 7.25) d) x = -2.5

d) a) (0, -10) b) ( -5 , 0 ) i ( 2 , 0 ) c) ( -1.5 , -12.25) d) x = -1.5

3.1.1 La gràfica de la funció està oberta cap amunt.


L'eix de simetria és la recta x=-3
El seu vèrtex és el punt ( -3 , 1 )
El punt de tall amb l'eix Y és y=10
No té cap punt de tall amb l'eix X.
3.2.2 a f , b g , c j, d f
3.2.3 a) (3,1) b) c) (-1,5) d) e) (2,5) f) g) (-1,9)

3.2.4 a) f ( x)  x2  8x  4  ( x  4)2  20 b) f ( x)  x2  6 x  5  ( x  3)2  4


c) f ( x)  x2  14 x  11  ( x  7)2  38 d) f ( x)  x2  6 x  17  ( x  3)2  8
e) f ( x)  x2  14 x  27  ( x  7)2  22 f) f ( x)  x2  10 x  9  ( x  5)2  34
2 2
 5  29  1  29
g) f ( x)  x 2  5 x  1   x    h) f ( x)  x 2  x  7   x   
 2 4  2 4
2
 3  19  11  1
i) f ( x)  x  3x  7   x   
2
j) f ( x)  x 2  11x  30   x   
 2 4  2 4
3.2.5 a) Mínim: (2,-13) b) c) Màxim (1,1) d) e) mínim (1/2, -5/4)
3.2.6 a) f ( x)  ( x  1)  2
2

b) V=(1,2), Punts de tall eix X :Cap. Punt tall eix Y: 3


c)

d) IR, 2,

b) a) f ( x)  ( x  3)2  9
b) V=(3,-9), Punts de tall eix X: 0, 6. Punt tall eix Y: 0
c)
d) IR,  9,
c) a) f ( x)  3( x  1)2  3
b) V=(-1,-3), Punts de tall eix X: -2, 0. Punt de tall eix Y: 0
c)

d) IR,  3,

d) a) f ( x)  ( x  3)2  13
b) V=(3,13). Punts de tall amb l'eix X: 3  13 . Punt de tall eix Y: 4
c)

d) IR,  ,13

e) a) f ( x)  2( x  1)2  1
b) V=(-1,1). Punts de tall eix X: Cap. Punt de tall eix Y: 3
c)

d) IR, 1,

f) a) f ( x)  4( x  3 / 2)2  10
b) V=(-3/2,10). Punts de tall eix X:  3 / 2  10 / 2 . Punt de tall eix Y: 1
c)
d) IR,  ,10

3.2.7 a) f ( x)  2 x 2 b) f ( x)  3( x  2)2 c) f ( x)  ( x  3)2

f) f ( x)   x  3  4
4
f ( x)   x 2  1 e) f ( x)  2( x  2)2  5
2
d)
9
3.2.8 a) f ( x)  x 2  4 x f ( x)  x 2  4 x  ( x  2)2  4  vèrtex  (2,4)
b) f ( x)  x 2  2 x  2 f ( x)  ( x  1)2  1  vèrtex  (1,1)
c) f ( x)  6 x  10  x 2 f ( x)  ( x  3)2  1  vèrtex  (3,1)
d) f ( x)  8  3x  x 2 f ( x)  ( x  3 / 2)2  41/ 4  vèrtex  (3 / 2 , 41/ 4)
e) f ( x)  2 x 2  8 x  9 f ( x)  2( x  2)2  1  vèrtex  (2 , 1 )
3.3.1
a) f ( x)  x 2  2 x  8
Punt de tall eix Y: -8, Punts de tall eix X: -2, 4; vèrtex: (1, -9)
b) f ( x)  x 2  2 x  15
Punt de tall eix Y: -15, Punts de tall eix X: -3, 5; vèrtex: (1, -16)
c) f ( x)  2 x 2  12 x  10
Punt de tall eix Y: 10, Punts de tall eix X: 1, 5; vèrtex: (3, -8)
d) f ( x)   x 2  2 x  15
Punt de tall eix Y: 15, Punts de tall eix X: -3, 5; vèrtex: (1, 16)
e) f ( x)  3x 2  3x  6
Punt de tall eix Y: 6, Punts de tall eix X: -1, 2; vèrtex: (1/2, 27/4)
3.3.2 f ( x)   x 2  2 x  4 , ( -3.24 , 0 ) , (1.24 , 0 )
3.3.3 a) f ( x)  0.07 x 2  1.5x  6 , f (18)  10.32  10.3 fa canasta.
Vídeo solució: Multimedia\will it hit the hoop\act3.mov

b) f ( x)  0.08x 2  1.7 x  5 , f (18)  9.68 no fa canasta.


Vídeo solució: Multimedia\will it hit the hoop\sequels\take2\act3.mov
3.4.1 a) f ( x)  2 x 2  x  5 b) f ( x)   x 2  4 x  3 c) f ( x)  3x 2  5x  1
3.5.1 a) ( 2 , 7 ) , ( -3 , -8 ) b) ( -1 ,4 ) , ( -4, -5) c) ( -4 , 5 ) , ( -2 , 1 )
3.5.2 ( -5 , -3 )
3.5.3 (0,-2), (4,2)

3.5.4 a) A=( -0.65 , -2.65 ) B=( 4.65 , 2.65 ) b) A=( 0.5 , 2 ) (un únic punt)
c) no existeix cap punt de tall. d) A=(-0.77 , 0.11) B=( 1.94 , 1.47)
e) A=(-3.79 , 15.37) B=( 0.79, 1.63) f) A=(-1 , 7) (un únic punt)
g) no tenen punts de tall.

3.5.5 a) A = ( -3.41 , -3.81) , B = ( 4.41 , 11.81 )

b) A = ( -2.79 , 3.79 ) , B = ( 1.79 , -0.79 )

c) A = ( -4.19 , -0.59 ) , B = (-1.31 , 0.84 )


d) A = ( 1.42 , -4.16 ) , B = ( 4.58 , 2.16 )

3.5.6 a) Paràbola: y  2 x 2  4 x  30 , Recta: y  2.5x  11


A  (3.19 , 3.04) , B  (6.44 , 27.09)
b) Paràbola: y   x 2  3x  28 , Recta: y  3x  5
A  (3.48 , 5.44) , B  (9.48 ,  33.44)

3.5.7 Paràbola: f ( x)  x 2  4 x  1 , Recta: g ( x)  x  1

Punts de tall: ( 0.438 , -0.562 ) , ( 4.562 , 3.562 )

3.5.8
a) (  0.791 ,  0.583 ) , ( 3.791 , 8.583 )
b) (  2.445 , 1.55 ) , ( 2.449 , 6.449 )
c) (  2 , 1 ) , ( 2 , 5 )

d) ( 0 , 8 ) , ( 4 , 0 )

3.5.10
a)   3.79 , 1.79  ,  0.79 ,  2.79 
b)   0.23 , 1.80  ,  2.89 , 9.09 
c)  0.8 ,  1.96 

d)   0.71 , 2.08 

3.5.11
a) f ( x)  0.5x  6 , g ( x)  x 2  5x  3 , P    1.5 , 6.75  , Q   6 , 3 
b) f ( x)  2 x  1 , g ( x)   x 2  2 x  5 , P    2.45 ,  5.9  , Q   2.45 , 3.9 
3.7.1

3.9.1 a) 4800 milers d’euros. c) x=150, B(150)=6400 d) 100 o 200 unitats


e) No tindrà pèrdues per 70  x  230
3.9.2 a) B(20)=0 b) x=50, B(50)=675 €
3.9.3 a) La funció creix en l’interval (0,3), decreix en l’interval (3,4) i (6,8) i és constant en
l’interval (4,6). S’arriba al rendiment màxim en t = 3. r(3) = –90 + 180 = 90. El
rendimient màxim és de 90.
b) El rendimient és 50 per a t = 1 y per a t = 8
3.9.4 a) x= 63 b) B(63)=36000€
3.9.5 a) T(2) = -6ºC b) T(0) = 44º (i és la temperatura màxima)
c) t=9/2=4.5. A les 4h 30’ es va arribar a la temperatura mínima.
T(4.5)=-12.25º C. L’interval de temperaturas va ser de -12.25º fins a 44ºC.
d) t=8 i t=1. A les 8 i a la 1 la temperatura va ser de 0ºC.
4.1.1 b i c
4.1.2 C
4.1.3 a) 4 b) 1/24 c) 16 d) 8
4.3.1 a) 2( x  5)( x  3)( x  2) b) 3( x  1)( x  1)( x  4) c)  2( x  4)( x  1) 2 ( x  3)
4.3.2 a)
Punt de tall amb l’eix Y: ( 0 , 9 )
Punts de tall amb l’eix X: -1, 1 i 3
40

20

-4 -2 2 4

-20

b)
Punt de tall amb l’eix Y: ( 0 , 18 )
Punts de tall amb l’eix X: -3, -1 i 3
100

75

50

25

-6 -4 -2 2 4 6
-25

-50

-75

c)
Punt de tall amb l’eix Y: ( 0 , 60 )
Punts de tall amb l’eix X: -5, -3, 1, 4
300

200

100

-6 -4 -2 2 4 6

-100

4.4.1
a) f ( x)  x 3  2 x 2  3x

b) f ( x)  2 x 4  x3  3x 2
c) f ( x)  x3  2 x 2  4 x  8

4.5.1 a) f ( x)  2( x  2)( x  1)( x  2)  2 x 3  2 x 2  8x  8


b) f ( x)  ( x  1)( x  3)( x  3)   x 3  x 2  9 x  9
c) f ( x)  2( x  1)( x  2)( x  3)( x  2)  2 x 4  4 x 3  14 x 2  16 x  24
d) f ( x)  ( x  1)( x  1)( x  3)( x  3)( x  5)  x 5  5x 4  10 x 3  50 x 2  9 x  45
4.5.2 i) C ii) A iii) D iv) B
4.5.3 a) a  1/ 3 b) -1 , 3
c)

4.6.1
4.6.2
4.7.1 Punt de tall amb l’eix Y: (0, -4) Punt de tall amb l’eix X: 4 / 3,0
4.7.2 f ( x)  3x  5
4.7.3 a) C= ( 0 , 4 ) b) A= ( 1 , 0 ) i B= ( 4 , 0 ) c) D = (2.5 , -2.25)
d) la recta x = 2.5
e)

4.7.4 f ( x)  3x 2  4 x  2
4.7.5 A = (-0.436 , 3.309 ) B = ( 3.4364 , -8.309 )
4.7.6 f ( x)  ( x  2) 2 ( x  1)( x  4)
4.7.7 f ( x)  3( x  2)( x  1)( x  4)  3x 3  15x 2  6 x  24
4.7.8 a) Exemples: f ( x)  x 2  1 , f ( x)  x 4  1
b) Exemples: f ( x)  ( x  4) 3 , f ( x)  ( x  4) 4
4.7.9 a = 5 b = 4
4.7.10 Punt de tall amb l’eix Y: ( 0 , 1 ) Punt de tall amb l’eix X: ( 1/2 , 0 )
4.7.11 f ( x)  3x  4
4.7.12 a) A=(0,1)
b) B1 = ( 0.29 , 0 ) B2 = ( 1.71 , 0 )
c) V=(1,-1)
d) x = 1
e)

4.7.13 f ( x)  x 2  2 x  1
4.7.14 A = ( -0.37 , -3.1 ) , B = ( 1.37 , 2.1 )
4.7.15 f ( x)  ( x  3)( x  2) 2 ( x  1)
4.7.16 f ( x)  2( x  1)( x  2)( x  3)
Punt de tall amb l’eix Y: ( 0 , 12 )
Punts de tall amb l’eix X: (1, 0) , (2, 0) i (-3,0)
4.7.17 f ( x)  2 x 3  2 x 2  12 x
4.7.18 a  5
4.7.19 -2 , 1 i 3
4.7.20 ( x  5)( x  3)( x  1)
4.7.21 -3, 1, 3, 5
4.7.22 ( x  3)( x  5)( x  2)  x 3  6 x 2  x  30
4.7.23  3( x  3)( x  5)( x  2)  3x 3  18x 2  3x  90
4.7.24 f ( x)  ( x  2) 2 ( x  3) 2 (exemple)
4.7.25 f ( x)  x( x  3)( x  1)( x  4)
4.7.26 k=3
4.7.27 k = 2, k = -1
4.7.28 a = -3 i b = 4
4.7.29 2.501
4.7.30 V = ( 0.333 , -0.666 )
4.7.31 A=(-2.303 , 5.908 ) B=(1.303 , -4.908 )
4.7.32 Eix Y: -48 , Eix X: { -2 , 2 , -4 }
4.7.33 f(x)=-1x2 + 3x + 2
4.7.34 P1=(-5,-22), P2=(-4,-14), P3=(2,-8)
4.7.35 f ( x)  3x 3  9 x 2  18x  24
5.5.3 a) 1/4 b) 2 c) -1
5.7.1 a) Asímptota horitzontal y  0 b) Asímptota horitzontal y  2
c) Asímptota obliqua y  2 x  6 d) Asímptota obliqua y  x  2
5.8.1 a)
03 3
a) x = -3 b) y  f (0)  2   0.6 c) x = -1 , x = 5
0  40 5 5

d) Grau Numerador = 1, Grau Denominador = 2, per tant lim f ( x)  0


x

b)
a) 2 i 4 b) 1.6 c) –5 i 1 d) tendeix a una asímptota horitzontal y = 1
e)
c)
a) 3 b) 9 c) –1
d) Tendeix a l’asímptota obliqua y = 6 x – 7
e)

d)
( x  1)( x  2)( x  3)
a) f ( x)  b) 1, -2 , 3 c) –1
( x  2)( x  3)

d) 2, -3 e) tendeix a l’asímptota obliqua y  x  3

f)
e)

( x  4)( x  3)( x  8)
f ( x) 
( x  1)( x  4)( x  6)

f)
( x  4)( x  7)
f ( x) 
( x  2)( x  3)( x  5)

g)
( x  1)( x  3)( x  7)
f ( x) 
( x  2)( x  5)
3 
6.3.1 a)  ,  b)  ,4  4, c)  ,5  3,
2 
d) 0, e)  ,3  2, f) [4,2)  (3,)
g)  ,2  3, h) 1, i) 0,    2 ,3   4 ,5   ...

1
6.3.2 a) x=10 b) x= 10 c) x=
4
6.3.3 Dom  2,3
6.4.1 a) P = (-2,-3), Ix = 7, Iy = -1.586

b) P = (-3,-5), Ix = 22, Iy = -3.268

c) P = (-5,-4), Ix = 11, Iy = -1.764


d) P = (0,-3), Ix = -9, Iy = -3

e) P = (4,0), Ix = 4, Iy = 2

f) P = (-2,2), Ix = -1.556, Iy = -2.243

g) P = (1,-4), Ix = -1, Iy = -1.172


h) P = (5/3,2), Ix = 0.333, Iy = -0.236

i) P = (-2,0), Ix = -2, Iy = -4

j) P = (5/3,-3), Ix = -1.333, Iy = -0.764

6.5.1
6.5.2 La primera és la part superior i la segona és la part inferior de la circumferència de
centre ( 0 , 0 ) i radi 3:

6.5.3 a) ( 2.562 , 1.562 ) b) ( -3.823 , 1.177 ) (-1.177 , 3.823 )

c) ( 1.293 , 2.707 ) , ( 2.707 , 1.293 )

6.5.4
49  x 2  25  x 2  3  49  x 2  25  x 2  3
Interpretem aquesta equació com la determinació dels punts de tall entre
f ( x)  49  x 2 i g ( x)  25  x 2  3

Que són dues semicircumferències, de radis 7 i 5, una d’elles desplaçada tres unitats cap
a dalt. Les dibuixem, i veiem que un dels punts de tall està amb x entre 4 i 5 i l’altre
amb el mateix valor però en negatiu.

5 3
Observació: Les solucions exactes són x    4.33
2
6.5.5

( -3.811 , 1.216 ) i ( 3.139 , 2.48 )

6.5.6

x  3.67 , x  3.67

 
6.5.7  3 , 2    1.73 , 2 
6.5.8

x= 4.34 , y=9.01
6.6.1

6.6.4 a) (2.69 , 2.39) b) ( 1 , 3 ) , ( 4 , 6 )

c) ( 3 , 1 ) d) ( 0.78 , -1.67 )
e) ( 1 , 1 )

6.7.1

6.7.2

6.7.3
6.7.4

6.7.5 a) D b) B c) A d) C
6.7.6 a) f ( x)  x  1  1 b) f ( x)   x  1 c) f ( x)   x  1
6.8.1 x = 23
6.8.2 x=7
6.8.3 x=1
6.8.4 x = 21
6.8.5

6.8.6 a) (-2 , -3 ) b) ( 2.5 , 0 ) c) (0 , -1 )


d)
6.8.7 a)

b) A = ( -0.36 , 2.28 )
6.8.8 a)

b) A = ( -1 , 3 ) B=(2,2)
6.9.1 a) x = 11 b) x = 10 c) x = 5 d) x = 21
6.9.2
6.9.3 a) (-8/3 , -5 ) b) ( 17/3 , 0 ) c) (0 , -2.17 )
d)

6.9.4
a)

b) ( 0 , -1 )

6.9.5

6.9.6
( 0 , 1 ) , ( 1 , 2 ) i ( 2, 3 )

6.9.7

6.10.1 a) x = 9 b) x = 18 c) x = -5 d) x = 12
6.10.2

6.10.3 a) (-2 , -2 ) b) ( -0.67 , 0 ) c) (0 , 0.45 )


d)
6.10.4 a)

b) A = ( 1.25 , 3.5 )
6.10.5 a)

b) A = ( -0.91 , 3.71 ) B = ( 1.91 , 2.29 )

7.1.1 i) C ii) B iii) A iv) E v) B


7.2.2
a) b)
c) d)

e) f)

7.2.3
7.3.1 a) f ( x)  3x d) f ( x)  1/ 4
x
b)
7.3.3 k  4 , a  5
7.4.1 a) 1 2 4 8 16
b) f (t )  2 t / 40

c) 32768
d) 800 segons (20 generacions)
e) 1200 segons (30 generacions)
7.5.1 a) a) ( 0 , -2) b) (1.585 , 0 ) c) f(1) = –1 d) y = -3

b) a) ( 0 , -4 ) b) (-3.0503 , 0 ) c) f(1) = –4.41 f(-1) = -3.305 d) y = -5

c) a) ( 0 , 2 ) b) no té c) f(1) = 4 f(-1) = 4/3 d) y = 1

d) a) ( 0 , 2 ) b) no té c) f(1) = 6 f(-1) = 2/3 d) y = 0

e) a) ( 0 , 0.5 ) b) no té c) f(1) = 1 f(-1) = 1/4 d) y = 0


f) a) ( 0 , 3 ) b) no té c) f(1) = 0.6 f(-1) = 15 d) y = 0

g) a) ( 0 , -1 ) b) no té c) f(1) = -2 f(-1) = -1/2 d) y = 0

h) a) ( 0 , -1 ) b) no té c) f(1) = -0.8 f(-1) = -1.25 d) y = 0

i) a) ( 0 , -1 ) b) ( 0.415 , 0 ) c) f(1) = 2 f(-1) = -5/2 d) y = -4

j) a) ( 0 , -3 ) b) no té c) f(1) = -2.4 f(-1) = -4.5 d) y = -2


k) a) ( 0 , 1 ) b) ( 0.203 , 0) c) f(1) = -7 f(-1) = 11/3 d) y = 5

7.5.2 a) b) c)

d) e) f)

g) h) i)
j) k) l)

m) n)

7.6.1 Representant gràficament les dues funcions observem que el seu punt de tall és
aproximadament x  1.4

Anem provant valors per a obtenir una millor aproximació:

x 2x  4  x
1.5 0.32
1 -1
1.25 -0.37
1.4 0.03
1.3 -0.23
1.35 -0.10

Per tant, x  1.35 ( x  1.38616698... )


7.7.1 f ( x)  3 x
7.7.2 f ( x)  (1.16993) x
7.7.3 f ( x)  (0.00316228) x
7.7.4 P=(-2, 2.04082), Q=(-1, 1.42857), R=(1.55, 0.57531) i S=( 3.9, 0.248818)
7.7.5 P=(0.595922, 2), Q=(-1.19184, 0.25), R=(-0.191844, 0.8) i S=(-1.97961 , 0.1)
7.7.6 a) x=0.203114, b) x=0.5, c) x=-1 d) x= 0.693147 e) x=1
7.7.7
a)
Any 1980 1981 1982 1985 1990 1995
Població 30 30.3015 30.606 31.5381 33.1551 34.855

b) Passats 69.3147 anys (és a dir, en l’any 2050)


c) Passats 79.3147 anys (és a dir, en l’any 2060)
7.7.9 A=(1.14903 , 1.104 )
7.7.10 A=(0.310826, 2.48084)
7.7.11 A=( 2.06524, 965.342)
8.1.2 30 cm
8.1.3 a) 4.2 m b) 12.5 cm.
8.2.1

8.3.1
a) f ( x)  log 6 x  1  5 b) f ( x)  log 5 x  1  3
Domini: x  1 , Recorregut: IR. Domini: x  1 , Recorregut: IR.

c) f ( x)  log 6 x  3  5 d) f ( x)  log 2 x  1  3

Imatge: x  3 , Recorregut: IR. Domini: x  1 , Recorregut: IR

e) f ( x)  log 4 x  1  4 f) f ( x)  log 5 x  1  1

Domini: x  1 , Recorregut: IR Domini: x  1 , Recorregut: IR

g) f ( x)  log 4 x  2  1 h) f ( x)  log 6 x  2  1
Domini: x  2 , Recorregut: IR Domini: x  2 , Recorregut: IR.

8.3.2 i) c ii) f iii) d iv) e v) b vi) a


10.1.5
a) b)

c) d)
e) f)

g) h)

11.2.1 a) f ( x)  3 x b) f ( x)  x  1  1 c) f ( x)   x  1
11.2.2
a)

b)
c)

d)

e)

f)
g)

h)

i)

j)
k)

l)

11.2.3 a)  f  g x   f g x   4  g x   4 
2
  2 4  x 2 si x  0
x 
no def. si x  0
Domini de definició: 0,

b) g  f x   g  f x   f ( x)  4  x 2

Domini de definició: 4  x 2  0    2 , 2 

c)
13.1.1
a) b)

eix Y: (0,3), eix X:  


3 , 0    1.73 , 0  eix Y: (0,-1), eix X:  5,0 
c) d)

eix Y: (0,1) , eix X:  


5 , 0    2.24 , 0  eix Y: (0,-3) , eix X:   1.26 , 0 
e)

eix Y: (0,-3.30) , eix X:   1.999 , 0 


13.1.2
a) b)

Eix Y: (0,4), Eix X: (-5.236,0), (-0.764,0) Eix Y: (-0.732,0) , Eix X: (-2,0)

c) d)

Eix Y: (0,-2), Eix X: (-3.464, 0), (3.464,0) Eix Y: (0,2), Eix X: (-1.585,0)

e)

Eix Y: (0,-6.14), Eix X: (3.389,0)


13.1.3 a) b)

Eix Y: ( 0 , 4 ) , Eix X: ( -2 , 0 ) , ( 2 , 0 ) Eix Y: ( 0 , -0.76 ) , Eix X: ( 4 , 0 )


c) d)

Eix Y: ( 0 , -1 ) , Eix X: ( 2.24 , 0 ) , ( -2.24 , 0 ) Eix Y: ( 0 , -2 ) , Eix X: ( 0.68 , 0 )


e)

Eix Y: ( 0 , -1.46 ) , Eix X: ( 4 , 0 )

También podría gustarte